Метод гаусса для системы линейных уравнений: Решение систем линейных уравнений методом Гаусса

Содержание

Страница не найдена — ПриМат

© 2012-2016: Нохум-Даниэль Блиндер (11), Анастасия Лозинская (10), Юлия Стерлянко (8), Денис Стехун (8), Валентин Малявко (8), Елизавета Савицкая (8), Игорь Любинский (8), Олег Шпинарев (7), Александр Базан (7), Анна Чалапчий (7), Константин Берков (7), Татьяна Корнилова (6), Влад Радзивил (6), Максим Швандт (6), Людмила Рыбальченко (6), Кирилл Волков (6), Марина Чайковская (5), Екатерина Шибаева (5), Мария Корень (5), Анна Семененко (5), Мария Илларионова (5), Сергей Черкес (5), Алиса Ворохта (5), Валерия Заверюха (5), Елизавета Снежинская (5), Вадим Покровский (5), Даниил Радковский (5), Влад Недомовный (5), Александр Онищенко (5), Андрей Метасов (5), Денис Базанов (5), Александр Ковальский (5), Александр Земсков (5), Святослав Волков (4), Иван Мясоедов (4), Владислав Стасюк (4), Алёна Гирняк (4), Николай Царев (4), Валентин Цушко (4), Павел Жуков (4), Роман Бронфен-Бова (4), Артём Романча (4), Анна Шохина (4), Иван Киреев (4), Никита Савко (4), Кондрат Воронов (4), Алина Зозуля (4), Иван Чеповский (4), Артем Рогулин (4), Игорь Чернега (4), Даниил Кубаренко (4), Ольга Денисова (4), Татьяна Осипенко (4), Яков Юсипенко (4), Ольга Слободянюк (4), Руслан Авсенин (4), Екатерина Фесенко (4), Дмитрий Заславский (4), Алина Малыхина (4), Андрей Лисовой (4), Полина Сорокина (4), Кирилл Демиденко (4), Дмитрий Стеценко (4), Александр Рапчинский (4), Таисия Ткачева (3), Владислав Бебик (3), Илья Бровко (3), Максим Носов (3), Филип Марченко (3), Катя Романцова (3), Илья Черноморец (3), Евгений Фищук (3), Анна Цивинская (3), Михаил Бутник (3), Станислав Чмиленко (3), Катя Писова (3), Дмитрий Дудник (3), Дарья Кваша (3), Игорь Стеблинский (3), Артем Чернобровкин (3), Виктор Булгаков (3), Дмитрий Мороз (3), Богдан Павлов (3), Игорь Вустянюк (3), Андрей Яроцкий (3), Лаура Казарян (3), Екатерина Мальчик (3), Анатолий Осецимский (3), Иван Дуков (3), Дмитрий Робакидзе (3), Вячеслав Зелинский (3), Данила Савчак (3), Дмитрий Воротов (3), Стефания Амамджян (3), Валерия Сиренко (3), Георгий Мартынюк (3), Виктор Иванов (3), Вячеслав Иванов (3), Валерия Ларикова (3), Евгений Радчин (3), Андрей Бойко (3), Милан Карагяур (3), Александр Димитриев (3), Иван Василевский (3), Руслан Масальский (3), Даниил Кулык (3), Стас Коциевский (3), Елизавета Севастьянова (3), Павел Бакалин (3), Антон Локтев (3), Андрей-Святозар Чернецкий (3), Николь Метри (3), Евелина Алексютенко (3), Константин Грешилов (3), Марина Кривошеева (3), Денис Куленюк (3), Константин Мысов (3), Мария Карьева (3), Константин Григорян (3), Колаев Демьян (3), Станислав Бондаренко (3), Ильдар Сабиров (3), Владимир Дроздин (3), Кирилл Сплошнов (3), Карина Миловская (3), Дмитрий Козачков (3), Мария Жаркая (3), Алёна Янишевская (3), Александра Рябова (3), Дмитрий Байков (3), Павел Загинайло (3), Томас Пасенченко (3), Виктория Крачилова (3), Надежда Кибакова (2), Майк Евгеньев (2), Евгений Колодин (2), Денис Карташов (2), Александр Довгань (2), Нина Хоробрых (2), Роман Гайдей (2), Антон Джашимов (2), Никита Репнин (2), Инна Литвиненко (2), Яна Юрковская (2), Гасан Мурадов (2), Богдан Подгорный (2), Алексей Никифоров (2), Настя Филипчук (2), Гук Алина (2), Михаил Абабин (2), Дмитрий Калинин (2), Бриткариу Ирина (2), Никита Шпилевский (2), Алексей Белоченко (2), Юлиана Боурош (2), Никита Семерня (2), Владимир Захаренко (2), Дмитрий Лозинский (2), Яна Колчинская (2), Юрий Олейник (2), Кирилл Бондаренко (2), Елена Шихова (2), Татьяна Таран (2), Наталья Федина (2), Настя Кондратюк (2), Никита Гербали (2), Сергей Запорожченко (2), Николай Козиний (2), Георгий Луценко (2), Владислав Гринькив (2), Александр Дяченко (2), Анна Неделева (2), Никита Строгуш (2), Настя Панько (2), Кирилл Веремьев (2), Даниил Мозгунов (2), Андрей Зиновьев (2), Андрей Данилов (2), Даниил Крутоголов (2), Наталия Писаревская (2), Дэвид Ли (2), Александр Коломеец (2), Александра Филистович (2), Евгений Рудницкий (2), Олег Сторожев (2), Евгения Максимова (2), Алексей Пожиленков (2), Юрий Молоканов (2), Даниил Кадочников (2), Александр Колаев (2), Александр Гутовский (2), Павел Мацалышенко (2), Таня Спичак (2), Радомир Сиденко (2), Владислав Шиманский (2), Илья Балицкий (2), Алина Гончарова (2), Владислав Шеванов (2), Андрей Сидоренко (2), Александр Мога (2),

Решение систем линейных уравнений методом Гаусса

Содержание:

Решение систем линейных уравнений методом Гаусса

  • Решение системы линейных уравнений Метод Гаусса (1) Одним из наиболее универсальных и эффективных методов решения систем линейной алгебры является метод Гаусса. i + ai2X2 H —— h alnxn = bi a22 x2 t ‘• * T a2n xn- » am2®2 + •• + ​​GmUn = bn • Где (t, j = 2, m) — новое значение коэффициента, Правильная часть получена после первого шага. Аналогичным образом исключают неизвестные X2 из всех уравнений системы, учитывая основной элемент <4UФ0, исключая первое и второе.

    Примеры решения и задачи с методическими указаниями

    Решение задачЛекции
    Сборник и задачникУчебник
    • Продолжайте этот процесс как можно больше. Если процесс приведения системы (1) к постепенной форме показывает нулевые уравнения, то есть уравнения вида 0 = 0, они отбрасываются. Если отображается уравнение вида 0 = aΦ0>, это указывает на несовместимость системы. Второй шаг (обратный) — это решение ступенчатой ​​системы. В общем, существует множество решений системы градуированных уравнений. В последнем уравнении этой системы первое неизвестное xb представлено оставшимися неизвестными (£ fc + 1, …, xn). Затем подставьте значение Xk в предпоследнее уравнение системы и выразите Xk- \ через a: n).

    Тогда найди Xk-2> … Примечания: 1. Если система ступеней представляет собой треугольник, то есть k = 7i, исходная система имеет единственное решение. Найти xn из последнего уравнения и из второго уравнения xn-1) из последнего далее в систему всех остальных неизвестных [xn — 2? ••• yXi). 2.

    Прибавьте произвольные значения к свободным неизвестным …, xn), получите бесконечное число решений для системы. Людмила Фирмаль

    На практике удобнее выполнять все базовые преобразования для строк, используя матрицу расширения, а не систему (1). Удобно, если коэффициент aj равен 1 (переместить уравнение на место или отделить обе стороны уравнения все ф 1). Пример: 1) Решить систему, используя метод Гаусса. 2x \ -x-2 + 3×3-5 # 4 = 1, X \ -X2-bx3 = 2 3xi-2×2-2hz-5×4 = 3, 7xi-5×2-9hz-10×4 = 8.

    ♦ В результате базового преобразования в расширенную матрицу системы / 2-1 3 «-5 1 \ 1-1-5 0 2 3 -2 -2 -5 3 \ 7-5-9-10 8 / 1 -1 -5 0 2 \ 0 1 13 -5 -3 0 1 13 -5 -3 х0 2 26-10-6 / ^ 1 -1 -5 0 2 \ 2-13 -5 1 3 -2 -2 -5 3 ^ 7-5-9-10 8J -1 О 1 Ах ах \ 0 O -5 0 ‘2 л 13-5-3 LLC O O O y Оригинальная система была уменьшена до ступенчатой системы. xi-x2-5xs = 2 x2 + 13 Гц + 5×4 = -3. Итак, общее решение системы: x2 = -5×4-13x-X \ = -5×4-8×3-1. 1, x2 = x3 = 0, x4 = 0. 2) Решить систему, используя метод Гаусса. — = О, -3, ♦ X1 + x2 + x3 = 3, 2xi + 3×2 + 3×3 = 7, 3X] + X2 + x3 = 5, 5xi-x2-. Xs = 3. ♦ Выполнять базовые преобразования в строках расширенной матрицы системы.

    / 11 1 3 \ / 11 1 3 \ / 1 1 1 3 \ / 1 1 1 3 \ 2337 010 1 0101 0101 31 15 ~ 0-2—2-4 ~ 0112 ~ 0011 \ 5 -1 -1 3 / \ 0 -b -6 -12 / \ 0 I 1 2 / \ 0 0 0 0 / Полученная матрица соответствует системе + X-2 + xs = 3, X-2 = 1 Xb = 1. Выполнение обратного хода приводит к £ 3 = 1, x2-1, Xj = 1.

    Решение методом Гаусса СЛАУ 3-5-ого порядка

    Метод Гаусса решения систем линейных алгебраических уравнений состоит в последовательном исключении неизвестных с помощью элементарных преобразований и сведении к верхней треугольной (ступенчатой или трапециевидной). После чего решают систему с конца к началу, подстановкой найденных решений.

    Рассмотрим примеры решения систем линейных уравнений методом Гаусса, взяв за справочник сборник задач Дубовика В.П., Юрика И.И. «Высшая математика».

    ————-

    Задача.

    Решить систему линейных алгебраических уравнений.

    1) (1. 189)

    2) (4. 195)

    3) (4. 198)

    Решение.

    1) Преобразуем исходную систему к ступенчатому виду. Для этого от второго уравнения вычтем первое, умноженное на 3, а от четвертого вычтем первое, умноженное на 4.

    В результате с третьего уравнения имеем Полученное значение подставляем в исходное уравнение для нахождения

    Полученные значения подставляем в первое уравнение

    Решением системы трех линейных уравнений будут следующие значения переменных

    2) Имеем систему трех уравнений с четырьмя неизвестными. В таких случаях одна переменная может быть свободна, а остальные будут выражаться через нее. Сведем систему к ступенчатому виду. Для этого от второго и третьего уравнения вычтем первое

    Из последних двух уравнений получаем идентичные решения

    .

    После подстановки в первое уравнение получим

    Данное уравнение связывает три переменные. Таким образом любая из переменных может быть выражена через две других

    Итак получим следующее решение

    3) Имеем разреженную систему линейных уравнений пятого порядка с пятью неизвестными. Сведем ее к ступенчатому виду. От второго уравнения вычтем первое и запишем в удобном для анализа виде

    Из второго уравнения находим, что . Подставляем значения во все нижние уравнения и переносим за знак равенства. Также поменяем второе с третьим уравнения местами

    Четвёртое и пятое уравнения эквивалентны. Выразим одну из переменных через другую

    Полученное значение подставим во второе уравнение и найдем

    Из первого уравнения определяем

    Решение системы уравнений следующее

    При вычислениях систем линейных алгебраических уравнений методом Гаусса нужно свезти систему линейных уравнений к ступенчатому виду. Для этого удобно записывать переменные под переменными, как в последнем примере, это ускорит решение. Остальное все зависит от матрицы, которую нужно решить и Ваших умений.

    ———————————————-

    Посмотреть материалы:

    Методы решения систем линейных уравнений. Метод Гауса.

    Линейными называются такие уравнения, в которых все переменные находятся в первой степени. Так же в высшей математике переменные могут обозначаться не просто x, y, z и т.д., а переменными с индексами —

    Решить систему уравнений означает найти такие значения переменных, при которых каждое уравнение системы превращается в верное равенство. Это правило применимо к любым системам уравнений с любым количеством неизвестных.

    Существует несколько методов решения систем линейных уравнений:

    • метод подстановки («школьный метод»), или, как его еще называют, методом исключения неизвестных;
    • метод почленного сложения (вычитания) уравнений системы;
    • метод Гаусса;
    • метод Крамера;
    • метод обратной матрицы.

    Рассмотрим некоторые из вышеуказанных методов.

    Pешение системы уравнений методом Гаусса

    Метод Гаусса является самым универсальным и эффективным и заключается в последовательном исключении переменных.

    Пример.

    Необходимо решить систему:

    Решение:

    Прямой ход.

    Представим исходную систему в следующем виде:


    На каждом этапе решения будем располагать с правой стороны расширенную матрицу,
    эквивалентную системе уравнений. Расширенная матрица представляет собой несколько иную
    форму записи исходной системы уравнений. Это позволит нам вести решение более наглядно.

    Исключим переменную x1 из последнего уравнения.

    Для удобства переведем систему уравнений в целые числа, для этого умножим коэффициенты
    первого уравнения на 3, а коэффициенты второго уравнения на -2:


    Умножим коэффициенты первого уравнения на -1.

    Обычно, данное преобразование системы выполняется в уме и не указывается при решении.


    Прибавим получившееся уравнение ко второму уравнению.

    Первое уравнение при этом не изменится в исходной системе.


    Обратный ход.

    Рассмотрим второе уравнение получившейся системы:

    Рассмотрим первое уравнение получившейся системы:

    Найдем значение переменной x1

    .

    Найдем значение переменной x

    2, подставив найденное значение x1.

    Ответ :

    Если решили построить дом, то проекты коттеджей (http://www.intexhome.ru/projects/) вам будут необходимы.


    Если материал был полезен, вы можете отправить донат или поделиться данным материалом в социальных сетях:

    Решение систем линейных уравнений методом Гаусса

    Теория

    Классическим методом решения систем линейных алгебраических уравнений является метод Гаусса (метод исключений Гаусса). Суть метода — это последовательное исключение неизвестных, т.е. когда с помощью элементарных преобразований система уравнений приводится к равносильной системе ступенчатого вида, из которой последовательно, начиная с последних переменных, находятся все остальные переменные.

    Матрица, составленная из все ai,j, называется

    основной матрицей системы. Если к этой матрице добавить вектор столбец, составленный из bi, то такая матрица называется расширенной матрицей системы.

    Теорема Кронекера-Капелли (условие совместности системы): системат совместна тогда и только тогда, ранг ее основной матрицы равен рангу ее расширенной матрицы.

    Алгоритм решения СЛАУ методом Гаусса подразделяется на два этапа:

    • На первом этапе (прямой ход) система приводится ступенчатой или треугольной форме. Вычтем из второго уравнения системы первое, умноженное на такое число, чтобы обнулился коэффициент при x1. Затем таким же образом вычтем первое уравнение из третьего, четвертого и т.д. Тогда исключаются все коэффициенты первого столбца, лежащие ниже главной диагонали. Затем при помощи второго уравнения исключим из третьего, четвертого и т.д. уравнений коэффициенты второго столбца. Последовательно продолжая этот процесс, исключим из матрицы все коэффициенты, лежащие ниже главной даигонали.
    • На втором этапе (обратный ход) выражаем все получившиеся базисные переменные через небазисные и построим фундаментальную систему решений. Если все переменные являются базисными, то получим единственное решение системы линейных уравнений. Эта процедура начинается с последнего уравнения, из которого выражают соответствующую базисную переменную (а она там всего одна) и подставляют в предыдущие уравнения, и так далее, поднимаясь по «ступенькам» наверх. Каждой строчке соответствует ровно одна базисная переменная, поэтому на каждом шаге, кроме последнего (самого верхнего), ситуация в точности повторяет случай последней строки.

    Методы решения невырожденных систем линейных алгебраических уравнений (СЛАУ) — по формулам Крамера, матричный способ. Метод Гаусса = метод последовательного исключения неизвестных при решения систем линейных алгебраических уравнений. Наличие решений.





    Адрес этой страницы (вложенность) в справочнике dpva.ru:
      главная страница  / / Техническая информация / / Математический справочник / / Решение уравнений и неравенств. Системы уравнений. Формулы. Методы. / / Системы уравнений. Понятие системы уравнений. Свойства систем уравнений. Линейные системы уравнений. Основные методы решения систем уравнений  / / Методы решения невырожденных систем линейных алгебраических уравнений (СЛАУ) — по формулам Крамера, матричный способ. Метод Гаусса = метод последовательного исключения неизвестных при решения систем линейных алгебраических уравнений. Наличие решений.

    Поделиться:   

    Методы решения невырожденных систем линейных алгебраических уравнений (СЛАУ) — по формулам Крамера, матричный способ.


    *Бабичева, Болдовская, Справочник по математике. СибАДИ, 2010. (классная книга)

    Метод Гаусса = метод последовательного исключения неизвестных при решения систем линейных алгебраических уравнений (СЛАУ). Наличие решений.


    *Бабичева, Болдовская, Справочник по математике. СибАДИ, 2010. (классная книга)
    Поиск в инженерном справочнике DPVA. Введите свой запрос:

    Дополнительная информация от Инженерного cправочника DPVA, а именно — другие подразделы данного раздела:

  • Системы уравнений. Понятие системы уравнений. Свойства систем уравнений. Линейные системы уравнений с двумя неизвестными. Основные методы решения систем уравнений
  • Системы линейных алгебраических уравнений (СЛАУ). Общий вид, матрица системы, СЛАУ в матричной форме, решение СЛАУ. Разновидности СЛАУ — совместная, несовместная, определенная, неопределенная, однородная, неоднородная… Обратная матрица и ее нахождение.
  • Вы сейчас здесь: Методы решения невырожденных систем линейных алгебраических уравнений (СЛАУ) — по формулам Крамера, матричный способ. Метод Гаусса = метод последовательного исключения неизвестных при решения систем линейных алгебраических уравнений. Наличие решений.
  • Собственные векторы, собственные значения матрицы и их нахождение. Характеристическое уравнение матрицы. Подпространство собственных векторов.
  • Поиск в инженерном справочнике DPVA. Введите свой запрос:
    Если Вы не обнаружили себя в списке поставщиков, заметили ошибку, или у Вас есть дополнительные численные данные для коллег по теме, сообщите , пожалуйста.
    Вложите в письмо ссылку на страницу с ошибкой, пожалуйста.
    Коды баннеров проекта DPVA.ru
    Начинка: KJR Publisiers

    Консультации и техническая
    поддержка сайта: Zavarka Team

    Проект является некоммерческим. Информация, представленная на сайте, не является официальной и предоставлена только в целях ознакомления. Владельцы сайта www.dpva.ru не несут никакой ответственности за риски, связанные с использованием информации, полученной с этого интернет-ресурса. Free xml sitemap generator

    НОУ ИНТУИТ | Лекция | Решение систем линейных уравнений

    Аннотация: В лекции рассматривается задача решения систем линейных уравнений. Приводятся необходимые определения и постановка задачи. Описывается последовательный и параллельный варианты одного из прямых методов решения линейных систем общего вида – метода Гаусса. Далее дается описание последовательного и параллельного алгоритмов, реализующих итерационный метод сопряженных градиентов

    Системы линейных уравнений возникают при решении ряда прикладных задач, описываемых дифференциальными, интегральными или системами нелинейных (трансцендентных) уравнений. Они могут появляться также в задачах математического программирования, статистической обработки данных, аппроксимации функций, при дискретизации краевых дифференциальных задач методом конечных разностей или методом конечных элементов и др.

    Матрицы коэффициентов систем линейных уравнений могут иметь различные структуру и свойства. Матрицы решаемых систем могут быть плотными, и их порядок может достигать несколько тысяч строк и столбцов. При решении многих задач могут появляться системы, обладающие симметричными положительно определенными ленточными матрицами с порядком в десятки тысяч и шириной ленты в несколько тысяч элементов. И, наконец, при рассмотрении большого ряда задач могут возникать системы линейных уравнений с разреженными матрицами с порядком в миллионы строк и столбцов.

    8.1. Постановка задачи

    Линейное уравнение с n неизвестными x0, x1, ѕ, xn-1 может быть определено при помощи выражения

    ( 8.1)
    где величины a0,a1,…,an-1 и b представляют собой постоянные значения.

    Множество из n линейных уравнений

    ( 8.2)
    называется системой линейных уравнений или линейной системой. В более кратком ( матричном ) виде система может представлена как

    где A=(ai,j) есть вещественная матрица размера nxn, а векторы b и x состоят из n элементов.

    Под задачей решения системы линейных уравнений для заданных матрицы А и вектора b обычно понимается нахождение значения вектора неизвестных x, при котором выполняются все уравнения системы.

    8.2. Алгоритм Гаусса

    Метод Гаусса – широко известный прямой алгоритм решения систем линейных уравнений, для которых матрицы коэффициентов являются плотными. Если система линейных уравнений невырожденна, то метод Гаусса гарантирует нахождение решения с погрешностью, определяемой точностью машинных вычислений. Основная идея метода состоит в приведении матрицы А посредством эквивалентных преобразований (не меняющих решение системы (8.2)) к треугольному виду, после чего значения искомых неизвестных могут быть получены непосредственно в явном виде.

    В подразделе дается общая характеристика метода Гаусса, достаточная для начального понимания алгоритма и позволяющая рассмотреть возможные способы параллельных вычислений при решении систем линейных уравнений. Более полное изложение алгоритма со строгим обсуждением вопросов точности получаемых решений может быть получено, например, в работах [ [ 6 ] , [ 22 ] , [ 47 ] ] и др.

    8.2.1. Последовательный алгоритм

    Метод Гаусса основывается на возможности выполнения преобразований линейных уравнений, которые не меняют при этом решения рассматриваемой системы (такие преобразования носят наименование эквивалентных ). К числу таких преобразований относятся:

    • умножение любого из уравнений на ненулевую константу;
    • перестановка уравнений;
    • прибавление к уравнению любого другого уравнения системы.

    Метод Гаусса включает последовательное выполнение двух этапов. На первом этапе – прямой ход метода Гаусса – исходная система линейных уравнений при помощи последовательного исключения неизвестных приводится к верхнему треугольному виду

    где матрица коэффициентов получаемой системы имеет вид

    На обратном ходе метода Гаусса (второй этап алгоритма) осуществляется определение значений неизвестных. Из последнего уравнения преобразованной системы может быть вычислено значение переменной xn-1, после этого из предпоследнего уравнения становится возможным определение переменной xn-2 и т.д.

    8.2.1.1. Прямой ход алгоритма Гаусса

    Прямой ход метода Гаусса состоит в последовательном исключении неизвестных в уравнениях решаемой системы линейных уравнений. На итерации i, 0<=i<n-1, метода производится исключение неизвестной i для всех уравнений с номерами k, большими i (т.е. i<k<=n-1 ). Для этого из этих уравнений осуществляется вычитание строки i, умноженной на константу ( aki/aii ), с тем чтобы результирующий коэффициент при неизвестной xi в строках оказался нулевым – все необходимые вычисления могут быть определены при помощи соотношений:

    (следует отметить, что аналогичные вычисления выполняются и над вектором b ).

    Поясним выполнение прямого хода метода Гаусса на примере системы линейных уравнений вида:

    На первой итерации производится исключение неизвестной x0 из второй и третьей строки. Для этого из этих строк нужно вычесть первую строку, умноженную соответственно на 2 и 1. После этих преобразований система уравнений принимает вид:

    В результате остается выполнить последнюю итерацию и исключить неизвестную x1 из третьего уравнения. Для этого необходимо вычесть вторую строку, и в окончательной форме система имеет следующий вид:

    На рис. 8.1 представлена общая схема состояния данных на i -й итерации прямого хода алгоритма Гаусса. Все коэффициенты при неизвестных, расположенные ниже главной диагонали и левее столбца i, уже являются нулевыми. На i -й итерации прямого хода метода Гаусса осуществляется обнуление коэффициентов столбца i, расположенных ниже главной диагонали, путем вычитания строки i, умноженной на нужную ненулевую константу. После проведения (n-1) подобной итерации матрица, определяющая систему линейных уравнений, становится приведенной к верхнему треугольному виду.


    Рис. 8.1. Итерация прямого хода алгоритма Гаусса

    При выполнении прямого хода метода Гаусса строка, которая используется для исключения неизвестных, носит наименование ведущей, а диагональный элемент ведущей строки называется ведущим элементом. Как можно заметить, выполнение вычислений является возможным только, если ведущий элемент имеет ненулевое значение. Более того, если ведущий элемент ai,i имеет малое значение, то деление и умножение строк на этот элемент может приводить к накоплению вычислительной погрешности и вычислительной неустойчивости алгоритма.

    Возможный способ избежать подобной проблемы может состоять в следующем: при выполнении каждой очередной итерации прямого хода метода Гаусса следует определить коэффициент с максимальным значением по абсолютной величине в столбце, соответствующем исключаемой неизвестной, т.е.

    и выбрать в качестве ведущей строку, в которой этот коэффициент располагается (данная схема выбора ведущего значения носит наименование метода главных элементов).

    Вычислительная сложность прямого хода алгоритма Гаусса с выбором ведущей строки имеет порядок O(n3).

    8.2.1.2. Обратный ход алгоритма Гаусса

    После приведения матрицы коэффициентов к верхнему треугольному виду становится возможным определение значений неизвестных. Из последнего уравнения преобразованной системы может быть вычислено значение переменной xn-1, после этого из предпоследнего уравнения становится возможным определение переменной xn-2 и т.д. В общем виде выполняемые вычисления при обратном ходе метода Гаусса могут быть представлены при помощи соотношений:

    Поясним, как и ранее, выполнение обратного хода метода Гаусса на примере рассмотренной в п. 8.2.1.1 системы линейных уравнений

    Из последнего уравнения системы можно определить, что неизвестная x2 имеет значение 3. В результате становится возможным разрешение второго уравнения и определение значение неизвестной x1=13, т.е.

    На последней итерации обратного хода метода Гаусса определяется значение неизвестной x0, равное -44.

    С учетом последующего параллельного выполнения можно отметить, что вычисление получаемых значений неизвестных может выполняться сразу во всех уравнениях системы (и эти действия могут выполняться в уравнениях одновременно и независимо друг от друга). Так, в рассматриваемом примере после определения значения неизвестной x2 система уравнений может быть приведена к виду

    Вычислительная сложность обратного хода алгоритма Гаусса составляет O(n2).

    Решающих систем с исключением Гаусса — College Algebra

    Цели обучения

    В этом разделе вы:

    • Запишите расширенную матрицу системы уравнений.
    • Напишите систему уравнений из расширенной матрицы.
    • Выполняет операции со строками в матрице.
    • Решите систему линейных уравнений с помощью матриц.

    Немецкий математик Карл Фридрих Гаусс (1777–1855).

    Карл Фридрих Гаусс жил в конце 18-го и начале 19-го веков, но он по-прежнему считается одним из самых плодовитых математиков в истории.Его вклад в математику и физику охватывает такие области, как алгебра, теория чисел, анализ, дифференциальная геометрия, астрономия и оптика. Его открытия в области теории матриц изменили способ работы математиков за последние два столетия.

    Мы впервые столкнулись с исключением Гаусса в системах линейных уравнений: две переменные. В этом разделе мы еще раз вернемся к этой технике решения систем, на этот раз с использованием матриц.

    Написание расширенной матрицы системы уравнений

    Матрица может служить средством представления и решения системы уравнений.Чтобы выразить систему в матричной форме, мы извлекаем коэффициенты переменных и констант, и они становятся элементами матрицы. Мы используем вертикальную линию, чтобы отделить записи коэффициентов от констант, по сути заменяя знаки равенства. Когда система написана в такой форме, мы называем ее расширенной матрицей.

    Например, рассмотрим следующую систему уравнений.

    Мы можем записать эту систему в виде расширенной матрицы:

    Мы также можем написать матрицу, содержащую только коэффициенты.Это называется матрицей коэффициентов.

    Система уравнений три на три, например

    имеет матрицу коэффициентов

    и представлена ​​расширенной матрицей

    Обратите внимание, что матрица написана так, что переменные выстраиваются в свои собственные столбцы: члены x идут в первый столбец, y -термы во втором столбце и z -термы в третьем столбце. Очень важно, чтобы каждое уравнение было написано в стандартной форме, чтобы переменные совпадали.Если в уравнении отсутствует член переменной, коэффициент равен 0.

    Для данной системы уравнений напишите расширенную матрицу.

    1. Запишите коэффициенты членов x как числа в первом столбце.
    2. Запишите коэффициенты членов y в виде чисел во втором столбце.
    3. Если есть z -термин, запишите коэффициенты в виде чисел в третьем столбце.
    4. Нарисуйте вертикальную линию и напишите константы справа от нее.

    Написание расширенной матрицы для системы уравнений

    Напишите расширенную матрицу для данной системы уравнений.

    Расширенная матрица отображает коэффициенты переменных и дополнительный столбец для констант.

    Запишите расширенную матрицу данной системы уравнений.

    Написание системы уравнений из расширенной матрицы

    Мы можем использовать расширенные матрицы, чтобы помочь нам решать системы уравнений, потому что они упрощают операции, когда системы не обременены переменными.Однако важно понимать, как переключаться между форматами, чтобы поиск решений был более плавным и интуитивно понятным. Здесь мы будем использовать информацию в расширенной матрице, чтобы записать систему уравнений в стандартной форме.

    Напишите систему уравнений из расширенной матрицы.

    Выполнение операций со строками в матрице

    Теперь, когда мы можем писать системы уравнений в форме расширенной матрицы, мы рассмотрим различные операции со строками, которые могут выполняться с матрицей, такие как сложение, умножение на константу и перестановка строк.

    Выполнение строковых операций над матрицей — это метод, который мы используем для решения системы уравнений. Чтобы решить систему уравнений, мы хотим преобразовать матрицу в форму строки-эшелона, в которой есть единицы вниз по главной диагонали от верхнего левого угла до нижнего правого угла и нули в каждой позиции ниже главной диагонали. как показано.

    Мы используем операции со строками, соответствующие операциям с уравнениями, чтобы получить новую матрицу, эквивалентную строкам в более простой форме.Вот рекомендации по получению формы рядного эшелона.

    1. В любой ненулевой строке первым ненулевым числом является 1. Оно называется ведущим 1.
    2. Любые нулевые строки помещаются внизу матрицы.
    3. Любая ведущая 1 находится ниже и правее предыдущей ведущей 1.
    4. Любой столбец, в котором в начале стоит 1, имеет нули во всех остальных позициях в столбце.

    Чтобы решить систему уравнений, мы можем выполнить следующие операции со строками, чтобы преобразовать матрицу коэффициентов в форму ряда строк и выполнить обратную подстановку, чтобы найти решение.

    1. Поменяйте местами ряды. (Обозначение 🙂
    2. Умножьте строку на константу. (Обозначение 🙂
    3. Добавить произведение одной строки на константу к другой строке. (Замечание:

    Каждая из строковых операций соответствует операциям, которые мы уже научились решать системы уравнений с тремя переменными. С помощью этих операций есть несколько ключевых ходов, которые быстро достигнут цели написания матрицы в виде эшелона строк. Чтобы получить матрицу в виде эшелона строк для поиска решений, мы используем метод исключения Гаусса, который использует операции со строками для получения 1 в качестве первой записи, чтобы строку 1 можно было использовать для преобразования оставшихся строк.

    Исключение по Гауссу

    Метод исключения Гаусса относится к стратегии, используемой для получения матрицы в виде строки-эшелона. Цель состоит в том, чтобы записать матрицу с номером 1 в качестве записи по главной диагонали и иметь все нули внизу.

    Первый шаг стратегии Гаусса включает получение 1 в качестве первой записи, так что строка 1 может использоваться для изменения строк ниже.

    Учитывая расширенную матрицу, выполните операции со строками для получения формы «строка-эшелон».

    1. Первое уравнение должно иметь старший коэффициент 1. Поменяйте местами строки или умножьте на константу, если необходимо.
    2. Используйте операции со строками, чтобы получить нули в первом столбце под первой записью 1.
    3. Используйте операции со строками, чтобы получить 1 в строке 2, столбец 2.
    4. Используйте операции со строками, чтобы получить нули в нижнем столбце 2, под записью 1.
    5. Используйте операции со строками, чтобы получить 1 в строке 3, столбец 3.
    6. Продолжайте этот процесс для всех строк, пока в каждой записи по главной диагонали не будет 1, а внизу будут только нули.
    7. Если какие-либо строки содержат все нули, поместите их внизу.
    Решение системы методом исключения Гаусса

    Решите данную систему методом исключения Гаусса.

    Решите данную систему методом исключения Гаусса.

    Использование исключения Гаусса для решения системы уравнений

    Используйте метод исключения Гаусса для решения данной системы уравнений.

    Решение зависимой системы

    Решите систему уравнений.

    Выполнение операций со строками в расширенной матрице 3 × 3 для получения формы Row-Echelon

    Выполняет строковые операции с заданной матрицей для получения формы «строка-эшелон».

    Запишите систему уравнений в виде ряда.

    Решение системы линейных уравнений с использованием матриц

    Мы видели, как написать систему уравнений с расширенной матрицей, а затем как использовать строковые операции и обратную подстановку для получения формы «строка-эшелон».Теперь мы перейдем на шаг дальше от строковой формы, чтобы решить систему линейных уравнений 3 на 3. Общая идея состоит в том, чтобы исключить все переменные, кроме одной, с помощью операций со строками, а затем выполнить обратную замену для поиска других переменных.

    Решение системы линейных уравнений с использованием матриц

    Решите систему линейных уравнений с помощью матриц.

    Решение зависимой системы линейных уравнений с использованием матриц

    Решите следующую систему линейных уравнений, используя матрицы.

    Решите систему, используя матрицы.

    Можно ли решить любую систему линейных уравнений методом исключения Гаусса?

    Да, система линейных уравнений любого размера может быть решена методом исключения Гаусса.

    Дана система уравнений, решите с помощью матриц с помощью калькулятора.

    1. Сохранить расширенную матрицу как матричную переменную
    2. Используйте функцию ref ( в калькуляторе, вызывая каждую матричную переменную по мере необходимости.

    Решение систем уравнений с матрицами с помощью калькулятора

    Решите систему уравнений.

    Применение матриц 2 × 2 к финансам

    Кэролайн инвестирует в общей сложности 12 000 фунтов стерлингов в две муниципальные облигации, одна из которых выплачивает 10,5% годовых, а другая — 12%. Годовой процент, полученный по двум инвестициям в прошлом году, составил 1335 фунтов стерлингов. Сколько было вложено по каждой ставке?

    Применение матриц 3 × 3 к финансам

    Ava инвестирует в общей сложности 10 000 фунтов стерлингов в три счета, один из которых платит 5% годовых, другой — 8%, а третий — 9%.Годовой процент, полученный по трем инвестициям в прошлом году, составил 770 евро. Сумма, вложенная под 9%, была вдвое больше, чем сумма, вложенная под 5%. Сколько было вложено по каждой ставке?

    У нас есть система трех уравнений с тремя переменными. Пусть будет сумма, вложенная под 5%, пусть будет сумма, вложенная под 8%, пусть будет сумма, вложенная под 9%. Таким образом,

    В качестве матрицы имеем

    Теперь мы выполняем исключение Гаусса, чтобы получить форму строки-эшелон.

    Третья строка сообщает usthus

    Вторая строка говорит нам, что подставляя мы получаем

    Первая строка говорит нам о замене и получаем

    Ответ: 3000 евро вложено под 5%, 1000 евро вложено под 8% и 6000 евро вложено под 9%.

    Небольшая обувная компания взяла ссуду в размере 1 500 000 фунтов стерлингов для расширения своих запасов. Часть денег была взята под 7%, часть — под 8%, часть — под 10%. Сумма займа под 10% в четыре раза превышала сумму займа под 7%, а годовая процентная ставка по всем трем займам составляла 130 500 евро. Используйте матрицы, чтобы найти сумму займа по каждой ставке.

    ? 150 000 при 7%, 750 000 фунтов стерлингов при 8%, 600 000 фунтов стерлингов при 10%

    Ключевые концепции

    • Расширенная матрица — это матрица, которая содержит коэффициенты и константы системы уравнений.См. (Рисунок).
    • Матрица, дополненная постоянным столбцом, может быть представлена ​​как исходная система уравнений. См. (Рисунок).
    • Операции со строками включают в себя умножение строки на константу, добавление одной строки к другой строке и замену строк местами.
    • Мы можем использовать метод исключения Гаусса для решения системы уравнений. См. (Рисунок), (Рисунок) и (Рисунок).
    • Операции со строками выполняются над матрицами для получения формы «строка-эшелон». См. (Рисунок).
    • Чтобы решить систему уравнений, запишите ее в форме расширенной матрицы.Выполните операции со строками, чтобы получить форму эшелона строк. Обратно-заменитель, чтобы найти решения. См. (Рисунок) и (Рисунок).
    • Калькулятор можно использовать для решения систем уравнений с использованием матриц. См. (Рисунок).
    • Многие реальные проблемы можно решить с помощью расширенных матриц. См. (Рисунок) и (Рисунок).

    Упражнения по разделам

    Словесный

    Можно ли записать любую систему линейных уравнений в виде расширенной матрицы? Объясните, почему да или почему нет. Объясните, как написать эту расширенную матрицу.

    Да. Для каждой строки коэффициенты переменных записываются поперек соответствующей строки и помещается вертикальная черта; затем константы помещаются справа от вертикальной полосы.

    Можно ли записать любую матрицу в виде системы линейных уравнений? Объясните, почему да или почему нет. Объясните, как написать эту систему уравнений.

    Есть только один правильный метод использования операций со строками в матрице? Попытайтесь объяснить две различные операции со строками, которые можно выполнить для расширенной матрицы

    .

    Нет, существует множество правильных методов использования строковых операций над матрицей.Есть два возможных способа: (1) Поменять местами строки 1 и 2. Затем (2) Разделить строку 1 на 9.

    Можно ли решить матрицу с нулевым элементом на диагонали? Объясните, почему да или почему нет. Что бы вы сделали, чтобы исправить ситуацию?

    Может ли матрица с 0 элементами для всей строки иметь одно решение? Объясните, почему да или почему нет.

    Нет. Матрица с 0 элементами для всей строки будет иметь либо ноль, либо бесконечно много решений.

    Алгебраические

    Для следующих упражнений напишите расширенную матрицу для линейной системы.

    Для следующих упражнений запишите линейную систему из расширенной матрицы.

    Для следующих упражнений решите систему методом исключения Гаусса.

    Расширения

    Для следующих упражнений используйте метод исключения Гаусса для решения системы.

    Реальные приложения

    Для следующих упражнений настройте расширенную матрицу, описывающую ситуацию, и найдите желаемое решение.

    Ежедневно в магазине кексов продается 5 000 кексов со вкусом шоколада и ванили. Если вкус шоколада в 3 раза популярнее, чем аромат ванили, сколько кексов продается в день?

    В конкурирующем магазине кексов ежедневно продаются кексы на сумму 4520 фунтов стерлингов.Шоколадные кексы стоят 2,25 евро, а кексы из красного бархата — 1,75 евро. Если общее количество кексов, проданных в день, составляет 2200, сколько штук каждого вкуса продается каждый день?

    860 красный бархат, 1340 шоколад

    Вы вложили 10 000 евро в два счета: один с простой процентной ставкой 3%, а другой — с процентной ставкой 2,5%. Если ваша общая сумма процентов по истечении одного года составила 283,50 фунтов стерлингов, какая сумма была на каждом счете по истечении года?

    Вы вложили 2300 евро на счет 1 и 2700 евро на счет 2.Если общая сумма процентов по истечении одного года составляет 254 евро, а на счете 2 процентная ставка в 1,5 раза выше, чем на счете 1, каковы процентные ставки? Предположим простые процентные ставки.

    4% на счет 1, 6% на счет 2

    Bikes’R’Us производит велосипеды по 250 фунтов стерлингов. Производитель обошелся в 180 фунтов стерлингов за велосипед плюс стартовый взнос в размере 3500 фунтов стерлингов. Через сколько проданных велосипедов производитель выйдет на уровень безубыточности?

    Крупный магазин бытовой техники рассматривает возможность приобретения пылесосов у небольшого производителя.Магазин сможет приобрести пылесосы по цене 86 фунтов стерлингов каждый, с оплатой доставки в размере 9 200 фунтов стерлингов, независимо от того, сколько пылесосов продано. Если магазин должен начать получать прибыль после продажи 230 единиц, сколько они должны взимать плату за пылесосы?

    Три самых популярных вкуса мороженого — это шоколад, клубника и ваниль, составляющие 83% вкусов, продаваемых в магазине мороженого. Если ваниль продается на 1% больше, чем в два раза больше клубники, а шоколад продается на 11% больше, чем ваниль, сколько в общем потреблении мороженого приходится на ароматы ванили, шоколада и клубники?

    В магазине мороженого растет спрос на три вкуса.В прошлом году банановое, тыквенное и мороженое с каменистой дорогой составили 12% от общего объема продаж мороженого. В этом году на те же три вида мороженого пришлось 16,9% продаж мороженого. Продажи по каменистой дороге выросли вдвое, продажи бананов увеличились на 50%, а продажи тыквы — на 20%. Если у мороженого по каменистой дороге было на один процент меньше продаж, чем у бананового мороженого, узнайте, какой процент продаж мороженого было произведено каждым отдельным мороженым в прошлом году.

    Банан — 3%, тыква — 7%, а каменистая дорога — 2%

    Пакет с ореховой смесью содержит кешью, фисташки и миндаль.Всего в сумке 1000 орехов, а миндаля на 100 меньше, чем фисташек. Кешью весит 3 г, фисташки — 4 г, миндаль — 5 г. Если мешок весит 3,7 кг, узнайте, сколько орехов каждого вида в нем.

    Пакет с ореховой смесью содержит кешью, фисташки и миндаль. Изначально в сумке было 900 орехов. Было съедено 30% миндаля, 20% кешью и 10% фисташек, и теперь в сумке осталось 770 орехов. Первоначально кешью было на 100 штук больше, чем миндаля.Для начала выясните, сколько орехов каждого типа было в пакете.

    100 миндальных орехов, 200 кешью, 600 фисташек

    Глоссарий

    дополненная матрица
    матрица коэффициентов, примыкающая к столбцу констант, разделенному вертикальной линией в скобках матрицы
    матрица коэффициентов
    матрица, содержащая только коэффициенты из системы уравнений
    Исключение по Гауссу
    с использованием элементарных операций со строками для получения матрицы в виде строки-эшелона
    главная диагональ
    записей из левого верхнего угла по диагонали в правый нижний угол квадратной матрицы
    рядная форма
    после выполнения операций со строками матричная форма, которая содержит единицы по главной диагонали и нули в каждом пробеле ниже диагонали
    эквивалент ряда
    две матрицы и эквивалентны строкам, если одна может быть получена из другой путем выполнения основных операций со строками
    строковые операции
    : добавление одной строки к другой строке, умножение строки на константу, перестановка строк и т. Д. С целью получения формы «строка-эшелон»

    Часть 6: Исключение по Гауссу.Исключение Гаусса — это алгоритм… | Авниш | Линейная алгебра

    Метод исключения Гаусса — это алгоритм решения системы линейных уравнений. Он назван в честь немецкого математика Карла Фридриха Гаусса.

    Карл Фридрих Гаусс

    Он аналогичен методу исключения, описанному ранее.

    Для выполнения метода исключения Гаусса:

    1. Создаем расширенную матрицу коэффициентов и констант данной системы линейных уравнений.
    2. Выбираем нашу pivot (это первый элемент по диагонали).Затем мы пытаемся уменьшить все элементы под ним (до «0»), используя pivot.

    Мы делаем это, выполняя два вида операций:

    a) Умножение сводной строки (строки сводного элемента) на скалярную величину и вычитание или добавление ее строк под ней.

    b) Перестановка строк (например, строка 2 заменяется строкой 3)

    Затем мы выбираем следующую точку поворота (следующий элемент по диагонали) и уменьшаем элементы под ней.

    3. Разбиваем расширенную матрицу обратно на строковое изображение и выполняем умножение с переменной матрицей.Получаем новые редуцированные уравнения.

    Мы решаем эти уравнения, чтобы получить значения неизвестных (переменных).

    Предполагая, что мы должны найти решение (я) следующей системы уравнений:

    4x + y = 9 → (1)

    2x-y = 3 → (2)

    5x-3y = 7 → ( 3)

    (пример «Одно уникальное решение» из Части 5)

    Шаг 1 (Создание расширенной матрицы):

    Для выполнения исключения Гаусса мы берем изображение строки (1), (2) и (3) . Это будет выглядеть следующим образом:

    Затем мы создаем расширенную матрицу для матрицы коэффициентов и постоянной матрицы.

    Единая матрица со значениями коэффициентов и констант, разделенных пунктирной линией

    Шаг 2 (Исключение):

    Шаг 2A:

    Принимая элемент в верхнем левом углу (первый элемент по диагонали) в качестве стержня, мы стремимся исключить ( уменьшите до «0») все элементы под ним. Другими словами, мы должны преобразовать каждый элемент в столбце 1 в «0», кроме pivot.

    Элемент поворота будет выделен красным цветом, а элементы, которые необходимо исключить, — синим.

    Итак, мы умножаем первую строку на скаляр 1/2 и вычитаем ее из второй строки.

    Элемент в строке 2 и столбце 1 исключается.

    Затем мы умножаем первую строку на скаляр 5/4 и вычитаем из третьей строки.

    Элемент в строке 3 и столбце 1 исключен.

    Теперь все элементы в первом столбце равны «0», кроме точки поворота.

    Шаг 2B:

    Теперь следующий элемент по диагонали (второй столбец второй строки) установлен как опорный, и мы стремимся удалить все элементы под ним.

    Pivot выделен красным.

    Итак, мы умножаем вторую строку на скаляр 17/6 и вычитаем ее из третьей строки.

    Элемент в строке 3 и столбце 2 исключен.

    Результат — верхняя треугольная матрица.

    Текущее состояние расширенной матрицы называется эшелоном строк формы .

    Шаг 3 (обратная подстановка):

    Теперь мы конвертируем форму эшелона строки обратно в изображение строки.

    У нас было аналогичное уравнение на этапе 1

    При умножении мы получаем:

    Мы составляем уравнения из этих

    4x + y = 9 → (4)

    -3y / 2 = -3/2 → (5)

    Решая (5) относительно «y», получаем:

    y = 1

    Теперь подставляем y = 1 в (4):

    4x + 1 = 9

    4x = 8

    x = 2

    Итак, мы получаем x = 2 и y = 1, именно то, что мы получили, когда решали через изображение строки и изображение столбца в Части 5.

    Теперь применим тот же алгоритм еще в двух случаях (бесконечно много решений и нет решения).

    Бесконечно много решений

    Мы возьмем тот же пример, что и в части 5. А именно:

    x + 2y = 4 → (6)

    2x + 4y = 8 → (7)

    Шаг 1 (Создание дополненного матрица):

    Строковое изображение (6) и (7) Расширенная матрица строчного изображения выше

    Шаг 2 (Исключение):

    Принятие первого диагонального элемента («1») в качестве опорного.

    Pivot выделен красным, и мы должны удалить все элементы под ним (синим). Чтобы исключить «2», мы дважды вычитаем строку 1 из строки 2 Теперь последняя строка полностью заполнена 0

    Мы больше не делаем поворота. так как исключать нечего.

    Шаг 3 (обратная подстановка):

    Мы преобразуем форму эшелона строк обратно в изображение строки:

    После этого мы умножаем ее и получаем новые уравнения

    x + 2y = 4 → (8)

    Уравнение (6) и уравнения (8) такие же, и у нас есть только одно уравнение после исключения, но два неизвестных («x» и «y»).

    Существует множество значений, которыми можно заменить x и y, чтобы удовлетворить (8).

    Нравится, x = 0 и y = 2. Подставляя в уравнение (8), получаем:

    0 + 2 (2) = 4

    4 = 4

    Или x = 1 и y = 1.5. Подставляя в уравнение (8), получаем:

    1 + 2 (1.5) = 4

    1 + 3 = 4

    4 = 4

    Таким образом, система уравнений (6) и (7) имеет бесконечно много решений.

    Нет решения

    Рассмотрение системы линейных уравнений следующим образом:

    x + y = 4 → (9)

    x + y = 8 → (10)

    xy = 0 → (11)

    Применение Гаусса Устранение.

    Шаг 1 (Создание расширенной матрицы):

    Изображение строки (9), (10) и (11) Расширенная матрица коэффициентов и констант

    Шаг 2 (Исключение):

    Принятие первого диагонального элемента в качестве точки поворота

    Мы выполняем следующие две операции:

    и получаем матрицу:

    У нас все еще нет формы эшелона строк (верхняя треугольная матрица).

    Итак, мы выполняем обмен строк (который также является вариантом на этапе исключения из метода исключения по Гауссу):

    Замена строки 3 на строку 2 Форма эшелона строки

    Шаг 3 (обратная подстановка):

    Форма ступени эшелона преобразована обратно в изображение строки

    Уравнения, которые мы получаем после умножения матриц выше:

    x + y = 4 → (12)

    -2y = -4 → (13)

    Решая уравнение (13) относительно «y», получаем:

    y = 2

    Подставляя y = 2 в уравнение (12), мы получаем:

    x + 2 = 4

    x = 2

    Чтобы подтвердить, что x = 2 и y = 2 — решение, мы подставляем их в систему уравнений i.е. (9), (10) и (11).

    Подставляя в (9), получаем:

    2 + 2 = 4

    4 = 4

    x = 2 и y = 2, удовлетворяет (9).

    Подставляя в (10), получаем:

    2 + 2 = 8

    4 ≠ 8, это не удовлетворяет (10).

    Следовательно, x = 2 и y = 2 не является решением (9), (10) и (11), и не существует решения этой системы линейных уравнений, как мы видели в прошлой статье.

    Одно решение

    Когда количество неизвестных (переменных) равно количеству уравнения в системе линейных уравнений.

    На примере (1), (2) и (3):

    4x + y = 9 → (1)

    2x-y = 3 → (2)

    5x-3y = 7 → (3)

    Имеется 2 неизвестных («x» и «y») и 3 уравнения ((1), (2) и (3)).

    Двух уравнений было бы достаточно для двух неизвестных.

    Бесконечно много решений

    Когда количество неизвестных превышает количество уравнений.

    В примере (6) и (7):

    x + 2y = 4 → (6)

    2x + 4y = 8 → (7)

    Есть 2 неизвестных («x» и «y») и 2 уравнения ((6) и (7)).

    Но если мы посмотрим дальше, то заметим, что (7) просто (6) умножается на 2. Следовательно, (6) и (7) являются одними и теми же уравнениями. Таким образом, количество уравнений равно 1.

    Нет решения

    Обычно, когда количество уравнений превышает количество неизвестных. У него нет решения.

    Но также необходимо, чтобы никакое значение не могло быть заменено на неизвестные, если удовлетворяется система линейных уравнений. Для этого нужно дважды проверить, удовлетворяет ли полученное решение всем уравнениям в системе.

    Гауссовское УСТРАНЕНИЕ: РЕШЕНИЕ СИСТЕМ ЛИНЕЙНЫХ УРАВНЕНИЙ: ПРИМЕРЫ И РЕШАЕМЫЕ ЗАДАЧИ: СТАРШАЯ ШКОЛА

    Содержимое этой страницы:


    Введение

    Система уравнений (линейная) — это группа (линейных) уравнений с различные неизвестные факторы. Вообще говоря, неизвестные факторы появляются в различных уравнениях.

    Уравнение с различными неизвестными факторами связывает их друг с другом.

    Решение системы состоит в нахождении значения неизвестных факторов способом, который проверяет — все уравнений, составляющих систему.

    • Если существует одно решение (одно значение для каждого неизвестного фактора), мы будем говорить, что система Согласованная независимая система (СНГ) .

    • Если есть различные решения (система имеет бесконечно много решений), мы говорим, что система является Согласованная зависимая система (CDS). .

    • Если решения нет, а это произойдет, если есть два или несколько уравнений, которые нельзя проверить одновременно, мы говорим, что это несовместимая система (IS) .Например, следующая система уравнений

      $$ \ begin {case} \ begin {array} {lcl} y & = & 0 \\ 2x + y & = & 0 \\ 2x + y & = & 2 \ end {array} \ end {cases} $ $

      несовместимо, потому что мы получаем решение x = 0 из второго уравнения и, из третьего, x = 1 .

    В этом разделе мы собираемся решать системы, используя метод исключения Гаусса , который заключается в простом выполнении элементарных операций в строке или столбце расширенной матрицы, чтобы получить свой эшелон из или его сокращенный эшелон форма (Гаусс-Иордан).


    Метод разрешения

    1. Применяем метод исключения Гаусса-Джордана : получаем уменьшенная форма эшелона строки из расширенной матрицы систему уравнений, выполняя элементарные операции в строках (или столбцах).

    2. Получив матрицу, мы применяем теорему Руше-Капелли для определения тип системы и получить решение (я), а именно:

      Пусть A · X = B будет системой из m линейных уравнений с n неизвестными множители, m и n натуральные числа (не ноль):

      • AX = B соответствует тогда и только тогда, когда,

        $$ ранг (A) = ранг (A | B) $$

      • AX = B является непротиворечивым независимым тогда и только тогда, когда,

        $$ ранг (A) = n = ранг (A | B) $$

    Примечание: Элементарные операции в строках или столбцах позволяют получить системы, эквивалентные исходной, но с формой, упрощающей получение решений (если они есть).Также есть более быстрые инструменты для выработки решений в СНГ, такие как правило Крамера.




    Система 1

    Показать решение

    Расширенная матрица системы

    того же размера, что и система (2×3). Вертикальная линия, отделяющая матричные коэффициенты от вектора независимых членов.

    Выполняем элементарные операции в строках для получения приведенной формы эшелона строк:

    Умножаем первую строку на 1/5 а вторую на 1/3

    Добавляем вторую строку с первой

    Вторую строку умножаем на 5/7

    Складываем первую строку со второй, умноженной на -2/5

    Эта последняя эквивалентная матрица представлена ​​в сокращенной форме эшелона строк. и это позволяет нам быстро увидеть рейтинг матрица коэффициентов и дополненная.

    Считаем ранги:

    По теореме Руше-Капелли система непротиворечива Независима. Полученная матрица представляет собой систему

    , который является решением исходной системы.



    Система 2

    Показать решение

    Расширенная матрица системы

    Проводим элементарные операции в строках для получения приведенных форма эшелона строки:

    Вторую строку умножаем на 1/2

    Добавляем первую строку со второй

    Умножаем первую строку на 1/3

    Эта последняя эквивалентная матрица находится в форме сокращенного эшелона строк и имеет нулевую строку, что означает, что строки в исходной системе линейно зависимы. (любой из них может быть получен умножением другого на скаляр, не равный нулю).

    Рассчитываем ранги

    По теореме Руше-Капелли система непротиворечива. Кроме того, он является зависимым, поскольку ранг (1) ниже количества неизвестных факторов (2).

    Полученная матрица представляет собой систему

    Решения



    Система 3

    Показать решение

    Расширенная матрица системы

    Выполняем элементарные операции в строках, чтобы получить сокращенную форму звена строк

    Мы меняем порядок строк (так у нас уже будет 1 в первой строке без необходимости умножать)

    Складываем вторую строку с первой, умноженной на 5 :

    Вторую строку умножаем на -1/15

    Эта последняя матрица имеет эшелонированную форму (не сокращена).

    Мы можем непосредственно заметить, что система несовместима, потому что у нас следующее равенство (вторая строка):

    $$ 0x + 0y = 1 $$

    , что невозможно равенство.

    Рассчитываем ранги матрицы коэффициентов и дополненной:

    По теореме Руше-Капелли система несовместна (решения нет). Полученная матрица представляет собой систему

    Система непоследовательна, потому что мы имеем невозможное равенство

    $$ 0 = 1 $$



    Система 4

    Показать решение

    Расширенная матрица системы

    (размер 3х4).

    Выполняем элементарные операции в строках, чтобы получить сокращенную форму звена строк

    Умножаем первую строку на 1/5

    Складываем вторую и третью строки с первой, умноженной на -2

    Вторую и третью строки умножаем на 5

    Складываем вторую строку с третьей, умноженной на -1

    Умножаем вторую строку на -1/10 и третью на 1/11

    Складываем первую строку со второй, умноженной на -2/5 , а третью со второй, умноженной на -1

    Третью строку умножаем на -11/5

    Эта последняя эквивалентная матрица находится в сокращенной форме эшелона строк (мы знаем ее, потому что это единичная матрица).Имея единичную матрицу, мы знаем, что это непротиворечивая независимая система, и можем получить единственное решение.

    Рассчитываем ранги

    По теореме Руше-Капелли система непротиворечива Независима. Полученная матрица представляет собой систему

    , который является решением системы.



    Система 5

    Показать решение

    Расширенная матрица системы

    Выполняем элементарные операции в строках, чтобы получить сокращенную форму звена строк

    Мы вычитаем первую строку из второй и добавляем третью строку с первой.

    Умножаем первую строку на 1/2 а вторую на 1/3

    Мы складываем первую строку со второй, умноженной на 1/2 , и третью строку со второй, умноженной на -2

    Третью строку умножаем на 1/3

    Складываем первую строку с третьей, умноженной на -3/2

    Эта последняя матрица представляет собой эшелонированную матрицу с сокращенной строкой (мы знаем это, потому что у нас есть единичная матрица).

    Рассчитываем ранги

    По теореме Руше-Капелли система непротиворечива Независима. Полученная матрица представляет собой систему



    Система 6

    Показать решение

    Расширенная матрица системы

    Выполняем элементарные операции в строках, чтобы получить сокращенную форму звена строк

    Умножаем вторую строку на -1/3 и третью на 1/4

    Складываем вторую и третью строки, умножив первую на -1

    Умножаем вторую строку на -3/4 и третью на -1

    Третью строку складываем со второй, умноженной на -1

    Эта последняя матрица имеет эшелонированную форму (но не сокращена) и мы не продолжаем выполнять операции по строкам, потому что можем видим, что последняя строка делает систему непоследовательной.Эта строка сообщает нам:

    $$ 0x + 0y + 0z = -1 $$

    И все это невозможное равенство.

    Фактически у нас есть ранги

    По теореме Руше-Капелли система несовместна.



    Система 7

    Показать решение

    Расширенная матрица системы

    Выполняем элементарные операции в строках, чтобы получить сокращенную форму звена строк

    Умножаем первую и вторую строки на 1/3 и на 1/6 соответственно

    Вторую строку вычитаем из третьей

    Добавляем первую строку со второй

    Третью строку умножаем на 3

    Эта последняя матрица находится в сокращенном эшелоне строк формы , поэтому мы можем легко вычислить ранги:

    Рассчитываем ранги

    По теореме Руше-Капелли система непротиворечива.Но он не является независимым, потому что количество неизвестных факторов (3) отличается от ранга. Полученная матрица представляет собой систему

    Решения



    Система 8

    Показать решение

    Расширенная матрица системы

    Примечание: перед тем, как мы начнем, отметим, что процедура будет таким же, как и до сих пор.Но у нас есть корни в матрице, а это значит, что операции в строках будут длинными и утомительно. Эта задача не очень интересна в дидактическом смысле, помимо расчетов.

    Проводим элементарные операции в строках, чтобы получить сокращенный рядный эшелон формы

    Мы складываем вторую строку с первой, умноженной на -√5 и, с третья, умноженная на -2/5

    Умножаем вторую строку на 1 / √5 и третью на 5/17

    К первой прибавляем третью и ко второй вычитаем третью

    Переписываем матрицу

    Вторую строку умножаем на (√5 / 5-5) -1

    Складываем первую строку со второй, умноженной на -5

    В этой последней матрице она (почти) в строке уменьшена форма эшелона (надо поменять второй и третий ряды так что это правда).Из последней матрицы получаем решения:



    Система 9

    Показать решение

    Расширенная матрица системы

    Выполняем элементарные операции в строках, чтобы получить сокращенную форму звена строк

    Складываем третью и четвертую строки, умножая первую на 3 и на -2 соответственно

    Складываем первую, вторую и третью строки с четвертой, умноженной на 2 , -3 и 5 соответственно

    Четвертую строку умножаем на -1 и меняем на вторую

    Умножаем третью и четвертую строки на 1/34 и -1/22 соответственно

    Вычитаем третью строку из четвертой

    Четвертую умножаем на -187/42

    К первой строке прибавляем третью умноженную на -13 и вторую на 8

    К первой строке прибавляем четвертую, умноженную на 5/34 , ко второй прибавляем первую, умноженную на 5/17 и к третьему добавляем первое, умноженное на -3/34

    По теореме Руше-Капелли система непротиворечива Независима, и решение равно



    Система 10

    Показать решение

    Расширенная матрица системы

    Примечание: Эта система была включена в цель отметить, что теория матрицы применима к комплексным числам.Единственное отличие от предыдущих систем в том, что теперь нам нужно действовать путем умножения и деления комплексные числа.

    Выполняем элементарные операции в строках для получения приведенной формы эшелона строк

    Умножаем вторую строку на 1/2 и меняем ее на первую

    Складываем вторую строку с первой, умноженной на — (1 + i)

    Вторую строку умножаем на

    Добавляем первую строку ко второй, умножаем на -i / 2

    Эта последняя матрица имеет сокращенную форму, потому что это единичная матрица.

    По теореме Руше-Капелли система непротиворечива Независима, и решение равно



    Matesfacil.com от J. Llopis под лицензией творческий Международная лицензия Commons Attribution-NonCommercial 4.0.


    Исключение Гаусса (Введение в линейные системы)

    Метод исключения Гаусса — это процедура решения систем линейных уравнений.Его можно описать как последовательность операций, выполняемых над соответствующей матрицей коэффициентов. Мы мотивируем исключение Гаусса и исключение Гаусса – Жордана несколькими примерами с упором на понимание операций со строками.

    Введение в системы линейных уравнений

    Система линейных уравнений с двумя переменными $ x $ и $ y $ имеет вид $$ \ begin {cases} ax + by = c \\ d x + ey = f \ end {ases} $$ где $ a, b , c, d, e, f $ заданы числа, такие как действительные или комплексные числа.Иногда система линейных уравнений также записывается с использованием индексов \ begin {equal} \ label {2by2sys} \ begin {cases} a_ {11} x + a_ {12} y = b_1 \\ a_ {21} x + a_ {22} y = b_2 \ end {case} \ end {формула}, чтобы уменьшить количество используемых букв. Для простоты давайте временно предположим, что коэффициенты являются ненулевыми действительными числами, и зададимся вопросом: сколько решений может быть у системы $ 2 \ times 2 $? Ключевая идея состоит в том, чтобы понять, что каждое линейное уравнение представляет собой линию в декартовой плоскости. Если мы рассмотрим возможные способы пересечения прямых на плоскости, мы приходим к выводу, что не должно быть решений, одно единственное решение или бесконечно много точек $ (x, y) $, которые решают систему.

    Пример . Определите, соответствует ли система линейных уравнений \ begin {уравнение} \ label {consys} \ begin {cases} 2x + 3y = 0 \\ 4x + 5y = 0. \ end {ases} \ end {Equation} не имеет решений, ровно одно решение или бесконечно много решений. Если мы умножим первое уравнение, а именно $ 2x + 3y = 0 $, на 2 и вычтем из второго уравнения $ 4x + 5y = 0 $, мы получим $ y = 0. $ Следовательно, решение является единственным и имеет вид $ (x , y) = (0,0). $

    Система линейных уравнений $ 3 \ times 3 $ имеет вид \ begin {equal} \ label {3by3sys} \ begin {cases} a_ {11} x + a_ {12} y + a_ {13} z = b_1 \\ a_ {21} x + a_ {22} y + a_ {23} z = b_2 \\ a_ {31} x + a_ {32} y + a_ {33} z = b_3 \\ \ end {case} \ end { уравнение}, где $ a_ {ij} $ и $ b_1, b_2, b_3 $ — числа, а $ x, y, z $ — переменные.Геометрически линейные уравнения с тремя переменными — это просто плоскости в трех измерениях. Итак, каковы различные типы наборов решений для системы? Рассматривая возможные способы пересечения трех плоскостей в трех измерениях, мы приходим к выводу, что не должно быть решений, одно единственное решение или бесконечно много точек $ (x, y, z) $, которые решают систему.

    Пример . Определите, соответствует ли система линейных уравнений \ begin {уравнение} \ label {linesysex1} \ begin {cases} x + 2y + 3z = 0 \\ 4x + 5y + 6z = 3 \\ 7x + 8y + 9z = 0 \ end { case} \ end {equal} не имеет решений, ровно одно решение или бесконечно много решений.Умножьте первое уравнение на $ -2 $ и добавьте ко второму уравнению, получив уравнение $ 2x + y = 3. $ Удалив $ z $ из первого и третьего уравнений, мы получим $ 4x + 2y = 0 $, умножив первое уравнение. на $ -3 $ и добавив к третьему уравнению. $ X $ и $ y $, удовлетворяющие системе, также должны удовлетворять системе \ begin {уравнение} \ begin {cases} 2x + y = 3 \\ 4x + 2y = 0. \ end {ases} \ end {Equation} Умножение первого уравнения на $ 2 $ дает $ 4x + 2y = 6. $ Обратите внимание, что нет $ x $ и $ y $, которые удовлетворяют как $ 4x + 2y = 6 $, так и $ 4x + 2у = 0.$ Таким образом, у системы нет решений; поэтому исходная система также не имеет решений.

    Пример . Для чисел $ a, b $ и $ c $ покажите, что система линейных уравнений $$ \ begin {cases} x + 2y + 3z = a \\ x + 3y + 8z = b \\ x + 2y + 2z = c \ end {case} $$ либо не имеет решений, либо ровно одно решение, либо бесконечно много решений. Мы выбираем сначала удалить $ x $ и получаем систему $$ \ begin {cases} -y-5z = a-b \\ z = a-c. \ end {ases} $$ Затем мы исключаем $ z $, получая $ y = -6a + b + 5c.$ Следовательно, единственное решение — $$ (x, y, z) = (10a-2b-7c, -6a + b + 5c, a-c).

    $

    Давайте рассмотрим набор линейных уравнений, который включает $ n $ неизвестных величин, представленных как $ x_1, x_2, \ ldots, x_n. $ Пусть $ a_ {ij} $ представляет число, которое является коэффициентом $ x_j $ в $ i $ -ое уравнение. Пусть даны числа $ b_1, b_2, \ ldots, b_m $. Система линейных уравнений уравнений \ begin {equal} \ label {syseq} \ begin {cases} a_ {11} x_1 + a_ {12} x_2 + \ cdots + a_ {1n} x_n = b_1 \\ a_ {21} x_1 + a_ {22} x_2 + \ cdots + a_ {2n} x_n = b_2 \\ \ hfill \ vdots \ hfill \\ a_ {m1} x_1 + a_ {m2} x_2 + \ cdots + a_ {mn} x_n = b_m \\ \ end {cases} \ end {Equation} называется системой одновременных линейных алгебраических уравнений .

    Решение этой системы — это упорядоченный набор из $ n $ чисел, который удовлетворяет каждому из операторов $ m $ в системе. Система линейных уравнений без решения называется несовместимой , а система по крайней мере с одним решением называется согласованной . Массив $$ \ left [\ begin {array} {l | l} \ begin {matrix} a_ {11} & a_ {12} & \ cdots & a_ {1n} \\ a_ {21} & a_ {22} & \ cdots & a_ {2n} \\ & & \ vdots \\ a_ {m1} & a_ {m2} & \ cdots & a_ {mn} \\ \ end {matrix} & \ begin {matrix} b_1 \\ b_2 \\ \ vdots \\ b_m \ end {matrix} \ end {array} \ right] $$ называется расширенной матрицей , соответствующей системе линейных уравнений.

    Например, система линейных уравнений выше оказалась непротиворечивой, а другая система линейных уравнений выше оказалась непоследовательной. Дополненные матрицы для этих систем следующие. $$ \ left [\ begin {array} {l | l} \ begin {matrix} 2 & 3 \\ 4 & 5 \ end {matrix} & \ begin {matrix} 0 \\ 0 \ end {matrix} \ end {массив} \ right] \ qquad \ text {and} \ qquad \ left [\ begin {array} {l | l} \ begin {matrix} 1 & 2 & 3 \\ 4 & 5 & 6 \\ 7 & 8 & 9 \ end {matrix} & \ begin {matrix} 0 \\ 3 \\ 0 \ end {matrix} \ end {array} \ right] $$

    Пример .Найдите все решения следующей системы линейных уравнений. \ begin {Equation} \ label {example: 2by3system} \ begin {cases} -150 x + 500y = z \\ 50x + 100y + z = 200 \ end {cases} \ end {equal} Данная система эквивалентна $ $ \ begin {case} -150 x + 500y-z = 0 \\ 50x + 100y + z = 200. \ end {ases} $$ Сложение этих уравнений дает $ -100x + 600y = 200. $ Поскольку у нас есть одно уравнение с двумя переменными, одна из переменных свободна. Мы решили позволить $ y $ быть свободным. Пусть $ y = t $ для произвольного числа $ t. $ Тогда, решая для $ x $, мы получаем $ -100x = 200y-600y $, или, что то же самое, $ x = -2 + 6t.$ Подставляя в исходную систему, находим $$ z = -150 (-2 + 6t) + 500t = 300-400t. $$ Следовательно, существует бесконечно много решений, которые можно представить в виде множества $$ \ {(x, y, z) \ mid x = -2 + 6t, y = t, z = 300-400t \ text { где $ t \ in \ mathbb {R} $} \}.

    $

    Другая расширенная матрица для этой системы линейных уравнений — $$ \ left [\ begin {array} {l | l} \ begin {matrix} -150 & 500 & -1 \\ 50 & 100 & 1 \ end {matrix} & \ begin {matrix} 0 \\ 200 \ end {matrix} \ end {array} \ right] $$ Как вы думаете, возможно ли решить, проверяя расширенную матрицу, соответствует ли соответствующая система линейных уравнений будет последовательным или непоследовательным?

    Операции со строками: решение систем линейных уравнений

    До сих пор в наших примерах мы видели системы линейных уравнений, не имеющие решений, единственное решение или, возможно, бесконечное число решений.Эти примеры предлагают следующее определение.

    Определение . Две системы линейных уравнений называются эквивалентными , если они имеют один и тот же набор решений.

    Пример . Найдите систему линейных уравнений с тремя неизвестными $ x, y, z $, решениями которой являются $ x = 6 + 5t $, $ y = 4 + 3t $, $ z = 2 + t $, где $ t $ произвольно. Мы хотим исключить $ t. $ Решение для $ t $ дает $ t = z-2. $ Путем подстановки $ x = 6 + 5 (z-2) $ и $ y = 4 + 3 (z-2). $ Таким образом, у нас есть система линейных уравнений $$ \ begin {cases} x-5z = -4 \\ y-3z = -2 \ end {ases} $$, которая имеет бесконечно много решений.

    Теорема . Системы линейных уравнений эквивалентны, если каждая может быть получена из другой с помощью одной или нескольких следующих операций.

    Поменяйте порядок уравнений.

    Умножьте (или разделите) одно уравнение на ненулевой скаляр.

    Добавьте одно уравнение, кратное одному, к другому.

    Проба . Если мы рассмотрим набор решений как геометрический объект, должно быть очень легко понять, что способ, которым мы пишем уравнения, представляющие объект, не меняет объект.Таким образом, должно быть очевидно, что изменение порядка уравнений не изменит решений системы линейных уравнений. Ни умножение (или деление) обеих частей уравнения на ненулевую константу.

    Пусть система $ A $ будет системой $ m \ times n $, представленной \ begin {equal} \ label {syseqA} \ begin {cases} a_ {11} x_1 + a_ {12} x_2 + \ cdots + a_ {1n} x_n = b_1 \\ \ hfill \ vdots \ hfill \\ a_ {i1} x_1 + a_ {i2} x_2 + \ cdots + a_ {in} x_n = b_i \\ \ hfill \ vdots \ hfill \\ a_ {m1} x_1 + a_ {m2} x_2 + \ cdots + a_ {mn} x_n = b_m & \\ \ end {cases} \ end {equal} Рассмотрим систему $ B $, полученную из системы $ A $ добавлением $ k $ умноженного на уравнение $ i $ к уравнение $ j $ следующим образом: \ begin {Equation} \ label {syseqB} \ begin {cases} a_ {11} x_1 + a_ {12} x_2 + \ cdots + a_ {1n} x_n = b_1 \\ \ qquad \ qquad \ vdots \ qquad \ qquad \ qquad \ vdots \\ (a_ {j1} + k a_ {i1}) x_1 + (a_ {j2} + k a_ {i2}) x_2 + \ cdots + (a_ {jn} + k a_ {in }) x_n = b_j + k b_i \\ \ qquad \ qquad \ vdots \ qquad \ qquad \ qquad \ vdots \\ a_ {m1} x_1 + a_ {m2} x_2 + \ cdots + a_ {mn} x_n = b_m \\ \ end {cases} \ end {Equation} Пусть $ S_A $ и $ S_B $ — множества решений систем $ A $ и $ B $ соответственно.Мы покажем $ S_A = S_B. $

    Пусть $ x_0 = (x_1, x_2, \ ldots, x_n) $ — решение системы $ A. $ Таким образом, $ x_0 $ удовлетворяет всем линейным уравнениям. Итак, $ x_0 $ удовлетворяет всем уравнениям в системе B, кроме, возможно, $ j $ -го уравнения. Работая с $ j $ -м уравнением в системе $ B $, находим

    \ begin {align} & (a_ {j1} + k a_ {i1}) x_1 + (a_ {j2} + k a_ {i2}) x_2 + \ cdots + (a_ {jn} + k a_ {in}) x_n \ \ & \ qquad = a_ {j1} x_1 + \ cdots + a_ {jn} x_n + k (a_ {i1} x_1 + \ cdots a_ {jn} x_n) \\ & \ qquad = b_j + k b_i.\ end {align}

    Это показывает, что $ x_0 $ также удовлетворяет $ j $ -му уравнению системы $ B. $ Поскольку $ x_0 $ удовлетворяет каждому уравнению, $ x_0 $ также является членом $ S_B. $ До сих пор мы показали $ S_A \ substeq S_B . $ И наоборот, предположим, что $ x_0 $ является решением любого уравнения второй системы линейных уравнений. Работая с $ j $ -м уравнением системы $ A $, находим,

    \ begin {align} & a_ {j1} x_1 + a_ {j2} x_2 + \ cdots + a_ {jn} x_n = b_j \ notag \\ \ Longleftrightarrow \ quad & a_ {j1} x_1 + (k a_ {i1} -ka_ {i1}) x_1 + \ cdots + a_ {jn} + (k a_ {i1} -ka_ {i1}) x_n = b_j \ notag \\ \ Longleftrightarrow \ quad & (a_ {j1} + k a_ {i1}) x_1 + (a_ {j2} + k a_ {i2}) x_2 + \ cdots + (a_ {jn} + k a_ {in}) x_n = b_j + k b_i.\ label {eqsys} \ end {align}

    Следовательно, поскольку $ x_0 $ удовлетворяет системе $ B $; и, таким образом, $ x_0 $ удовлетворяет $ j $ -му уравнению системы $ A. $ Следовательно, $ S_A = S_B $, как и нужно.

    Прорабатывая детали следующих двух примеров, обратите внимание на то, как используются операции в Row Operations.

    Пример . Пусть $ a, b $ и $ c $ — константы. Решите систему линейных уравнений $$ \ begin {cases} y + z = a \\ x + z = b \\ x + y = c. \ end {ases} $$ Исключая $ z $ из первого и второго уравнений, получаем систему $$ \ begin {ases} xy = ba \\ x + y = c \ end {ases} $$ Решение для $ x $ дает $ x = (c + ba) / 2.$ Используя исходную систему, мы находим, что $ y $ и $ z $ равны $$ y = cx = c- (c + ba) / 2 = (c-b + a) / 2 $$ $$ z = ay = a — (c-b + a) / 2 = (a + bc) / 2 $$ Следовательно, решение системы: $$ (x, y, z) = \ left (\ frac {c + ba} {2 }, \ frac {a + cb} {2}, \ frac {a + bc} {2} \ right).

    $

    Пример . Найдите наименьшее натуральное число $ C $ такое, что $ x, y, z $ являются целыми числами и удовлетворяют системе линейных уравнений уравнений $$ \ begin {cases} 2x + y = C \\ 3y + z = C \\ x + 4z = C \ end {cases} $$ Умножьте третье уравнение на $ -2 $ и добавьте к первому уравнению, получив $ y-8z = -C.$ Умножая второе уравнение на 8 и прибавляя к $ y-8z = -C $, получаем $ 25y = 7C. $ Решая для $ y $, получаем $ y = (7/25) C. $ Подстановкой $ x = (9/25) C $ и $ z = (4/25) C. $ Следовательно, 25 — наименьшее целое число $ C $ такое, что $ x, y, z $ являются целыми числами и решает систему.

    Исключение по Гауссу

    Мы будем решать систему линейных уравнений, используя элементарные операции со строками над матрицами, используя процедуру, известную как Гауссово исключение . Набор решений будет набором векторов.

    Определение . Следующие операции вместе известны как операции элементарной строки .

    (1) Поменяйте местами два ряда.

    (2) Умножьте строку на ненулевой скаляр.

    (3) Добавить строку из другой строки, кратную строке.

    Из операций со строками очевидно, что применение элементарных операций со строками к системе линейных уравнений приводит к эквивалентной системе. Редукция системы линейных уравнений при сохранении множества решений — чрезвычайно полезная идея, которую мы будем активно развивать.

    Например, мы обнаруживаем, что система линейных уравнений \ begin {equal} \ label {refex} \ begin {cases} x + 2y + z = 3 \\ 2x + 5y-z = -4 \\ 3x-2y- z = 5 \ end {case} \ qquad \ qquad \ begin {cases} x = 2 \\ y = -1 \\ z = 3 \ end {case} \ end {уравнение} эквивалентны. Мы покажем, как применить элементарные операции со строками, чтобы получить систему справа. В то время как система слева может быть заданной системой линейных уравнений, система справа решена.

    Определение .Считается, что матрица находится в эшелоне строки в форме , если она удовлетворяет всем следующим условиям.

    (1) Все строки с хотя бы одним ненулевым коэффициентом находятся над любыми строками со всеми нулями.

    (2) Первое ненулевое число слева (называемое ведущим коэффициентом ) ненулевой строки всегда находится строго справа от ведущего коэффициента строки над ней.

    (3) Все записи в столбце под ведущим коэффициентом нулевые.

    Далее, матрица называется сокращенной формой эшелона строк , если она находится в форме эшелона строк и выполняется дополнительное условие: каждый ведущий коэффициент равен 1 и является единственной ненулевой записью в своем столбце.

    Например, расширенные матрицы для системы линейных уравнений выше: $$ \ begin {bmatrix} \ begin {array} {ccc | c} 1 & 2 & 1 & 3 \\ 2 & 5 & -1 & -4 \\ 3 & -2 & -1 & 5 \ end {array} \ end {bmatrix} \ qquad \ qquad \ begin {bmatrix} \ begin {array} {ccc | c} 1 & 0 & 0 & 2 \\ 0 & 1 & 0 & -1 \\ 0 & 0 & 1 & 3 \ end {array} \ end {bmatrix}.$$ Проверив эти четыре условия, мы увидим, что матрица справа имеет вид сокращенного ряда строк. Для получения дополнительных примеров рассмотрим следующие матрицы. $$ A = \ begin {bmatrix} 0 & 5 \\ 2 & 3 \ end {bmatrix} \ qquad B = \ begin {bmatrix} 1 & 0 \\ 1 & 0 \ end {bmatrix} \ qquad C = \ begin {bmatrix} 0 & 0 \\ 0 & 1 \ end {bmatrix} $$ $$ D = \ begin {bmatrix} 0 & 1 \\ 0 & 0 \ end {bmatrix} \ qquad E = \ begin {bmatrix} 1 & 0 \\ 0 & 1 \ end {bmatrix} \ qquad F = \ begin {bmatrix} 1 & 2 \\ 0 & 1 \ end {bmatrix} $$ Обратите внимание, что матрицы $ D $ и $ E $ находятся в уменьшенной строке форма эшелона, а остальные нет.

    Пример . Используйте метод исключения Гаусса-Жордана для решения системы линейных уравнений. $$ \ begin {cases} x + 2y + z = 3 \\ 2x + 5y-z = -4 \\ 3x-2y-z = 5 \ end {ases} $$ Используя операции со строками в расширенной матрице, мы получаем уменьшенная форма рядного эшелона.

    \ begin {align *} & \ begin {bmatrix} \ begin {array} {ccc | c} 1 и 2 и 1 и 3 \\ 2 и 5 и -1 и -4 \\ 3 и -2 и -1 и 5 \ end {массив} \ end {bmatrix} \ begin {array} {c} \ stackrel {\ longrightarrow} {\ scriptstyle -2R_1 + R_2} \\ \ stackrel {\ longrightarrow} {\ scriptstyle -3R_1 + R_3} \ end {массив} \ begin {bmatrix} \ begin {array} {ccc | c} 1 и 2 и 1 и 3 \\ 0 & 1 & -3 & -10 \\ 0 & -8 & -4 & -4 \ end {массив} \ end {bmatrix} \\ & \ qquad \ qquad \ begin {array} {c} \ stackrel {\ longrightarrow} {\ scriptstyle 8 R_2 + R_3} \ end {массив} \ begin {bmatrix} \ begin {array} {ccc | c} 1 и 2 и 1 и 3 \\ 0 & 1 & -3 & -10 \\ 0 & 0 & -28 & -84 \ end {массив} \ end {bmatrix} \ begin {array} {c} \ stackrel {\ longrightarrow} {\ scriptstyle — \ frac {1} {28} R_3} \ end {массив} \ begin {bmatrix} \ begin {array} {ccc | c} 1 и 2 и 1 и 3 \\ 0 & 1 & -3 & -10 \\ 0 и 0 и 1 и 3 \ end {массив} \ end {bmatrix} \\ & \ qquad \ qquad \ begin {array} {c} \ stackrel {\ longrightarrow} {\ scriptstyle 3R_3 + R_2} \\ \ stackrel {\ longrightarrow} {\ scriptstyle -R_3 + R_1} \ end {массив} \ begin {bmatrix} \ begin {array} {ccc | c} 1 и 2 и 0 и 0 \\ 0 & 1 & 0 & -1 \\ 0 и 0 и 1 и 3 \ end {массив} \ end {bmatrix} \ begin {array} {c} \ stackrel {\ longrightarrow} {\ scriptstyle -2R_2 + R_1} \ end {массив} \ begin {bmatrix} \ begin {array} {ccc | c} 1 & 0 & 0 & 2 \\ 0 & 1 & 0 & -1 \\ 0 и 0 и 1 и 3 \ end {массив} \ end {bmatrix}.\ end {align *}

    Следовательно, единственное решение — $ x = 2 $, $ y = -1 $ и $ z = 3. $

    Пример . Используйте метод исключения Гаусса-Жордана для решения следующей системы линейных уравнений. \ begin {уравнение} \ label {gjee1} \ begin {cases} x + y -2z + 4t = 5 \\ 2x + 2y-3z + t = 3 \\ 3x + 3y-4z-2t = 1 \ end {case } \ end {уравнение} Используя операции со строками над расширенной матрицей, мы получаем сокращенную форму эшелона строк.

    \ begin {align *} \ begin {bmatrix} \ begin {array} {cccc | c} 1 и 1 и -2 и 4 и 5 \\ 2 и 2 и -3 и 1 и 3 \\ 3 и 3 и -4 и -2 и 1 \ end {массив} \ end {bmatrix} & \ begin {array} {c} \ stackrel {\ longrightarrow} {\ scriptstyle -2R_1 + R_2} \\ \ stackrel {\ longrightarrow} {\ scriptstyle -3R_1 + R_3} \ end {массив} \ begin {bmatrix} \ begin {array} {cccc | c} 1 и 1 и -2 и 4 и 5 \\ 0 & 0 & 1 & -7 & -7 \\ 0 & 0 & 2 & -14 & -14 \ end {массив} \ end {bmatrix} \\ & \ begin {array} {c} \ stackrel {\ longrightarrow} {\ scriptstyle -2R_2 + R_3} \\ \ stackrel {\ longrightarrow} {\ scriptstyle 2R_2 + R_1} \ end {массив} \ begin {bmatrix} \ begin {array} {cccc | c} 1 и 1 и 0 и -10 и -9 \\ 0 & 0 & 1 & -7 & -7 \\ 0 & 0 & 0 & 0 & 0 \ end {массив} \ end {bmatrix} \ end {align *}

    Эта система эквивалентна $$ \ begin {ases} x + y-10t = -9 \\ z-7t = -7 \ end {cases} \ quad \ text {или, проще говоря} \ quad \ begin {cases}……………………………………………………………………………………………………………………………………………………………………………… х = -9-у + 10т \\ г = -7 + 7т.4 \ mid x = -9-y + 10t, y = s, z = -7 + 7t, w = t, \ text {for} s, t \ in \ mathbb {R} \}. $$ — это набор решений.

    Пример . Используйте метод исключения Гаусса-Жордана для решения следующей системы линейных уравнений. $$ \ begin {case} x_1 + x_2-2x_3 + 3x_4 = 4 \\ 2x_1 + 3x_2 + 3x_3-x_4 = 3 \\ 5 x_1 + 7 x_2 + 4 x_3 + x_4 = 5 \ end {cases} $$ Использование строки операций над расширенной матрицей мы получаем приведенную форму эшелона строк.

    \ begin {align *} \ begin {bmatrix} \ begin {array} {cccc | c} 1 и 1 и -2 и 3 и 4 \\ 2 и 3 и 3 и -1 и 3 \\ 5 и 7 и 4 и 1 и 5 \ end {массив} \ end {bmatrix} & \ begin {array} {c} \ stackrel {\ longrightarrow} {\ scriptstyle -2R_1 + R_2} \\ \ stackrel {\ longrightarrow} {\ scriptstyle -5R_1 + R_3} \ end {массив} \ begin {bmatrix} \ begin {array} {cccc | c} 1 и 1 и -2 и 3 и 4 \\ 0 и 1 и 7 и -7 и -5 \\ 0 и 2 и 14 и -14 и 15 \ end {массив} \ end {bmatrix} \\ & \ begin {массив} {c} \ stackrel {\ longrightarrow} {\ scriptstyle -2R_2 + R_3} \ end {массив} \ begin {bmatrix} \ begin {array} {cccc | c} 1 и 1 и -2 и 3 и 4 \\ 0 и 1 и 7 и -7 и -5 \\ 0 & 0 & 0 & 0 & 25 \ end {массив} \ end {bmatrix} \ end {align *}

    Обратите внимание, что последняя строка соответствует уравнению $ 0x_1 + 0x_2 + 0x_3 + 0x_4 = 25.2 $ будет таким многочленом. Используя данные точки, мы настраиваем систему и решаем.

    \ begin {align *} & \ left [\ begin {array} {c | c} \ begin {matrix} а & б & с \\ а & 2b & 4c \\ a, 3b и 9c \ end {matrix} & \ begin {matrix} -1 \ 3 \ 13 \ end {матрица} \ end {array} \ right] \ begin {array} {c} \ stackrel {\ longrightarrow} {\ scriptstyle -R_1 + R_2} \\ \ stackrel {\ longrightarrow} {\ scriptstyle -R_1 + R_3} \ end {массив} \ left [\ begin {array} {c | c} \ begin {matrix} а & б & с \\ 0 & b & 3c \\ 0 и 2b и 8c \ end {matrix} & \ begin {matrix} 1 \ 4 \ 14 \ end {матрица} \ end {array} \ right] \\ & \ qquad \ qquad \ begin {array} {c} \ stackrel {\ longrightarrow} {\ scriptstyle -2R_2 + R_3} \ end {массив} \ left [\ begin {array} {c | c} \ begin {matrix} а & б & с \\ 0 & b & 3c \\ 0 и 0 и 2c \ end {matrix} & \ begin {matrix} -1 \ 4 \ 6 \ end {matrix} \ end {array} \ right] \ begin {array} {c} \ stackrel {\ longrightarrow} {\ scriptstyle \ frac {1} {2} R_3} \ end {массив} \ left [\ begin {array} {c | c} \ begin {matrix} а & б & с \\ 0 & b & 3c \\ 0 & 0 & c \ end {matrix} & \ begin {matrix} -1 \ 4 \ 3 \ end {матрица} \ end {array} \ right] \\ & \ qquad \ qquad \ begin {array} {c} \ stackrel {\ longrightarrow} {\ scriptstyle -3R_3 + R_2} \ end {массив} \ left [\ begin {array} {c | c} \ begin {matrix} а & b & c \\ 0 & b & 0 \\ 0 & 0 & c \ end {matrix} & \ begin {matrix} -1 \ -5 \ 3 \ end {матрица} \ end {array} \ right] \ begin {array} {c} \ stackrel {\ longrightarrow} {\ scriptstyle -R_2 + R_1} \ end {массив} \ left [\ begin {array} {c | c} \ begin {matrix} а & 0 & с \\ 0 & b & 0 \\ 0 & 0 & c \ end {matrix} & \ begin {matrix} 4 \ -5 \ 3 \ end {матрица} \ end {array} \ right] \\ & \ qquad \ qquad \ begin {array} {c} \ stackrel {\ longrightarrow} {\ scriptstyle -R_3 + R_1} \ end {массив} \ left [\ begin {array} {c | c} \ begin {matrix} а & 0 & 0 \\ 0 & b & 0 \\ 0 & 0 & c \ end {matrix} & \ begin {matrix} 1 \ -5 \ 3 \ end {матрица} \ end {array} \ right].2-5 лет + 1. $

    Упражнения по системе линейных уравнений

    Упражнение . Убедитесь, что $ (2,3, -1) $ является решением системы линейных уравнений.

    • $ \ begin {cases} x + 2y + z = 7 \\ xy = -1 \\ 4x + y + z = 10 \ end {cases} $
    • $ \ begin {cases} x + y = 5 \ \ xz = 3 \\ y + z = 2 \ end {cases} $

    Упражнение . Убедитесь, что каждая тройка вида $ (7-2k, 8 + 6k, k) $ является решением системы линейных уравнений.

    • $ \ begin {cases} x_1 + 2x_3 = 7 \\ x_2-6x_3 = 8 \ end {cases} $
    • $ \ begin {cases} 2x_1 + 4x_3 = 14 \\ x_1 + 3x_2-16x_3 = 31 \ end {кейсы}

    Упражнение .У следующих систем нет решений, ровно одно решение или бесконечно много решений? Обосновать ответ.

    • $ \ begin {case} x + y = 1 \\ x + 2y = 1 \ end {cases} $
    • $ \ begin {cases} 3x + y = 1 \\ y = -2 \ end {case} $
    • $ \ begin {case} x + y = 1 \\ 2x + 2y = 2 \ end {cases} $
    • $ \ begin {case} 3x + y = 2 \\ 6x + 2y = 4 \ end { case} $

    Упражнение . Нарисуйте график каждого уравнения системы линейных уравнений и решите, нет ли у него решений, ровно одно решение или бесконечно много решений.

    • $ \ begin {case} x + y = 2 \\ 2x + 3y = 0 \ end {cases} $
    • $ \ begin {cases} -x + 3y = 2 \\ 2x-6y = -4 \ end {кейсы}

    Упражнение . Найдите решения, если таковые имеются, следующей системы линейных уравнений без использования матриц. Если уравнение имеет более одного решения, напишите общее решение.

    • $ \ begin {case} 3x-2y = 7 \\ 2x + y = 15 \ end {cases} $
    • $ \ begin {case} x + y = 7 \\ 3x + 4y = 12 \ end {случаях } $
    • $ \ begin {cases} 2x_1-3x_2 = 1 \\ 4x_1-6x_2 = -2 \\ x_1 + x_2 = 1 \ end {cases} $
    • $ \ begin {cases} 2x_1-5x_2 = 12 \ end {case} $
    • $ \ begin {cases} i x_1-3ix_2 = 1 \\ (2 + i) x_1-x_2 = -1 \ end {cases} $
    • $ \ begin {cases} (1 + i ) x_1-2ix_2 = 2 \\ 2x_1-3i x_2 = 4-3i \ end {cases} $

    Упражнение .Если возможно, найдите точки пересечения.

    • $ x-4y = 11 $ и 7x-2y = 9 $
    • $ x-4y + 3z = 11 $, 7x-2y-z = -1 $, 7x-2y + z = -2 $

    Упражнение . Пусть $ u = (1,1,2, -1) $ и $ v = (1,1,1,0). $ Для каких скаляров $ a $ и $ b $ верно, что $ a u + bv $ это решение следующей системы?

    • $ \ begin {cases} 4x-2y-zw = 1 \\ x + 3y-2z-2w = 2 \ end {cases} $
    • $ \ begin {cases} x-4y-z-2w = 4 \ \ 7x + y-5z-2w = 12 \ end {cases} $

    Упражнение .Найдите все решения, если таковые существуют, следующей системы линейных уравнений.

    • $ \ begin {case} xy = 1 \\ 2x = 4 \ end {cases} $
    • $ \ begin {case} 2x + 3y-z = 19 \\ 3x-2y + 3z = 7 \ end {случаях } $
    • $ \ begin {case} x-3y = 2 \\ -2x + 6y = -4 \ end {cases} $
    • $ \ begin {cases} x-y + 2z-2w = 1 \\ 2x + y + 3w = 4 \\ 2x + y + 3w = 6 \ end {cases} $

    Упражнение . Найдите все решения, если таковые существуют, для следующей системы линейных уравнений.

    • $ \ begin {case} 2x-z = -1 \\ x + yz = 0 \\ 2x-y + 2z = 3 \ end {cases} $
    • $ \ begin {cases} 2x-3y + 2z = 1 \\ x-6y + z = 2 \\ -x-3y-z = 1 \ end {cases} $

    Упражнение .Найдите все решения, если таковые существуют, для следующей системы линейных уравнений.

    • $ \ begin {cases} x_1 + 2x_2-x_3 = 0 \\ 2x_1 + 5x_2 + 5x_3 = 0 \\ x_1 + 4x_2 + 7x_3 = 0 \\ x_1 + 3x_2 + 3x_3 = 0 \ end {cases} $
    • $ \ begin {case} x + y + z + w = ​​1 \\ 2x-2y + z + 2w = 3 \\ 2x + 6y + 3z + 2w = 1 \\ 5x-3y + 3z + 5w = 8 \ end {кейсы}

    Упражнение . При каких значениях константы $ k $ у систем нет решения, ровно одно решение или бесконечное множество решений.

    • $ \ begin {case} x + y = 1 \\ 3x + 3y = k \ end {cases} $
    • $ \ begin {case} x + ky = 1 \\ 2x-y = k \ end {случаях } $

    Упражнение .2 + 2} + \ frac {c} {2x-1}.

    $

    Упражнение . Пусть $ a $ и $ b $ — произвольные постоянные. Найдите все решения системы линейных уравнений.

    • $ \ begin {case} x + 2y = a \\ 3x + 5y = b \ end {cases} $
    • $ \ begin {cases} a x + 2y = 1 \\ 3x + by = 4 \ end { case} $

    Упражнение .
    Пусть $ a $ и $ b $ — произвольные постоянные. Найдите все решения системы линейных уравнений.

    • $ \ begin {case} x + 2y + 3z = a \\ x + 3y + 8z = b \\ x + 2y + 2z = c \ end {cases} $
    • $ \ begin {cases} x-2y + 4z = a \\ x-3y + 5z = b \\ x-2y + 6z = c \ end {cases} $

    Упражнение .Система линейных уравнений, все постоянные члены которой равны нулю, называется однородной системой
    .

    • Покажите, что в однородной системе всегда есть хотя бы одно решение.
    • Приведите примеры, показывающие, что однородная система может иметь более одного решения или только одно решение.

    Упражнение . Напишите систему, соответствующую каждой расширенной матрице.

    • $ \ left [\ begin {array} {ccc | c} 1 & 2 & 1 & 1 \\ 0 & 4 & 0 & 1 \\ 0 & 0 & 3 & 3 \ end {array} \ right] $
    • $ \ left [\ begin {array} {cccc | c} 1 & 0 & 0 & 0 & 0 \ end {array} \ right] $
    • $ \ left [\ begin {array} {ccc | c} 1 & 0 & 0 & 0 \\ 0 & 1 & 0 & 0 \\ 0 & 0 & 1 & 0 \ end {array} \ right] $
    • $ \ left [\ begin {array} {c | c} 2 & 1 \\ 0 & 1 \\ 1 & 2 \ end {array} \ right] $
    • $ \ left [\ begin {array} {c | c} 1 & 1 \\ 1 & 0 \ end {массив } \ right] $
    • $ \ left [\ begin {array} {cccc | c} 1 & 1 & 1 & 1 & 0 \ end {array} \ right] $

    Упражнение .Какие из следующих матриц представлены в виде сокращенного ряда строк?

    • $ \ begin {bmatrix} 0 & 1 & 2 & 0 & 3 \\ 0 & 0 & 0 & 1 & 0 \\ 0 & 0 & 0 & 0 & 0 \ end {bmatrix} $
    • $ \ begin {bmatrix} 1 & 0 & 0 & 2 & 1 \\ 0 & 1 & 0 & 0 & 0 \\ 0 & 0 & 1 & 0 & 0 \ end {bmatrix} $
    • $ \ begin {bmatrix} 1 & 1 & 1 \\ 0 & 0 & 1 \\ 0 & 0 & 0 \\ 0 & 0 & 0 \ end {bmatrix} $
    • $ \ begin {bmatrix} 0 & 0 & 1 \\ 0 & 0 & 1 \\ 0 & 0 & 1 \\ \ end {bmatrix} $

    Упражнение .Какие из следующих матриц представлены в виде сокращенного ряда строк?

    • $ \ begin {bmatrix} 0 & 0 & 0 & 1 \\ 0 & 0 & 0 & 0 \\ \ end {bmatrix} $
    • $ \ begin {bmatrix} 0 & 0 & 0 \\ 0 & 0 & 0 \\ \ end {bmatrix} $
    • $ \ begin {bmatrix} 1 & 0 & 0 & 3 & 1 \\ 0 & 0 & 0 & 1 & 1 & 1 \\ 0 & 0 & 0 & 0 & 1 \ end {bmatrix} $
    • $ \ begin {bmatrix} 1 & 0 & 2 & 1 \\ 0 & 1 & 0 & 1 \\ 0 & 0 & 1 & 0 \\ 0 & 0 & 0 & 0 \ end {bmatrix } $

    Упражнение .Найти все решения системы линейных уравнений методом исключения Гаусса-Жордана.

    • $ \ begin {cases} x_2 + 2x_4 + 3x_5 = 0 \\ 4x_4 + 8x_5 = 0 \ end {cases} $
    • $ \ begin {cases} 3x + 4y = 0 \\ -2x + 7y = 0 \ конец {кейсы} $

    Упражнение . Найти все решения системы линейных уравнений методом исключения Гаусса-Жордана.

    • $ \ begin {case} x_4 + 2x_5-x_6 = 2 \\ x_1 + 2x_2 + x_5-x_6 = 0 \\ x_1 + 2x_2 + 2x_3-x_5 + x_6 = 2 \ end {cases} $
    • $ \ begin {case} 3x_1 + 3x_2-4x_3 + x_4 = 2 \\ x_1 + x_2-x_3-x_4 = 5 \ end {cases} $
    • $ \ begin {cases} 2 x_1 + 3x_2 + 4x_3 = 6-x_4 \\ 3x_1 -2x_2-x_4 = 1 + 4x_3 \\ 3x_1 + 3x_3 + x_4 = 4-x_2 \\ x_2 + x_3-4x_4 = -3-4x_1 \ end {cases} $
    • $ \ begin {cases} 4x_1 + 3x_2 + 2x_3 -x_4 = 4 \\ 5x_1 + 4x_2 + 3x_3-x_4 = 4 \\ -2x_1-2x_2-x_3 + 2x_4 = -3 \\ 11x_1 + 6x_2 + 4x_3 + x_4 = 11 \ end {case} $
    • $ \ begin {case} 2x_1-3x_2 + x_3 = 5 \\ x_1 + x_2-x_3 = 3 \\ 4x_1-x_2-x_3 = 1 \ end {cases} $
    • $ \ begin {cases} x_1-x_2 = 4 \\ 2x_1 + x_2 = 7 \\ 5x_1-2x_2 = 19 \ end {cases} $

    Упражнение .Найдите все матрицы размером $ 4 \ times 1 $ в приведенной форме эшелона строк.

    Упражнение . Сколько существует типов матриц размером $ 3 \ times 2 $ в приведенной строчной форме? Сколько существует типов матриц размером $ 2 \ times 3 $ в приведенной строчной форме?

    Упражнение . Опишите возможные формы сокращенного эшелона строк для матрицы с двумя строками и двумя столбцами. Опишите возможные формы сокращенного эшелона строк для матрицы с тремя строками и тремя столбцами.

    Упражнение .Найдите многочлен степени 3, график которого проходит через точки
    $ (0,1) $, $ (1,0) $, $ (- 1,0) $ и $ (2, -15). $ Нарисуйте эскиз график этой кубики.

    Упражнение . Найдите многочлен степени 4, график которого проходит через точки $ (1,1) $, $ (2, -1) $, $ (3, -59) $ и $ (- 2, -29). $ Sketch график этой квартики.

    Упражнение . Найдите значения $ k $, если они есть, для которых система имеет: только одно решение, никаких решений, бесконечное количество решений.2-2) x_3 = a \ end {cases} $$ будет иметь: бесконечно много решений, никаких решений, ровно одно решение.

    Упражнение . Используйте метод исключения Гаусса-Жордана для решения системы линейных уравнений $$ \ left \ {\ begin {array} {lll} a x_1 + b x_2 & & = r \ quad (a \ neq 0) \\ c x_1 + d x_2 & & = s \ quad (e \ neq 0) \\ & e x_3 + f x_4 & = t \\ & g x_3 + h x_4 & = u \\ \ end {array} \ right. $$ Условия состояния для $ a, b, c, d, e, f, g $ и $ h $, которые гарантируют уникальное решение.2 + a x + by + c = 0 $ окружности, проходящей через следующие точки.

    • $ (- 2,1) $, (5,0) $ и (4,1) $
    • $ (1,1) $, (5, -3) $ и $ (- 3, -3) $

    Как завершить решение этой однородной системы линейных уравнений с помощью исключения Гаусса?

    Итак, я собираюсь решить эту проблему, используя исключение Гаусса-Джордана, а не просто исключение Гаусса. По сути, это то же самое, за исключением того, что вы получаете $ 0 $ выше всех первых $ 1 $.

    Во-первых, поскольку это однородная система уравнений, вам не нужно использовать расширенную матрицу.Символы $ 0 $ в правой части строки останутся неизменными независимо от того, какую операцию строки вы используете.

    $ \ left (\ begin {matrix} 1 и 1 и 1 и 0 \\ 1 и 1 и 2 и -1 \\ 2 и 2 и 1 и 1 \\ -1 и -1 и 1 и -2 \\ \ end {matrix} \ right) $ $ \ begin {matrix} \\ Р_2-Р_1 \\ Р_3-2Р_1 \\ R_4 + R_1 \\ \ end {матрица} $ $ \ left (\ begin {matrix} 1 и 1 и 1 и 0 \\ 0 & 0 & 1 & -1 \\ 0 & 0 & -1 & 1 \\ 0 & 0 & 2 & -2 \\ \ end {matrix} \ right) $ $ \ begin {matrix} R_1-R_2 \\ \\ R_3 + R_2 \\ Р_4-2Р_2 \\ \ end {матрица} $ $ \ left (\ begin {matrix} 1 и 1 и 0 и 1 \\ 0 & 0 & 1 & -1 \\ 0 & 0 & 0 & 0 \\ 0 & 0 & 0 & 0 \\ \ end {matrix} \ right) $

    Для краткости я выполнил сразу несколько операций со строками, но это тот же ответ, что и вы.Взяв ваш ответ и разделив $ R_2 $ на $ 3 $, вычтите $ R_2 $ из $ R_1 $, и вы получите это решение Гаусса-Джордана.

    Каждая строка представляет собой уравнение, равное 0 $, потому что это однородная система уравнений. Первый столбец матрицы представляет собой представление переменных $ x_1 $ в каждом уравнении, второй столбец — это переменные $ x_2 $, третий — $ x_3 $, а четвертый — $ x_4 $. Первые $ 1 $ — это стержневые элементы матрицы, и они обычно выбираются в качестве зависимых переменных, но не обязательно.

    Следующим шагом в решении этой проблемы является возврат в нормальный формат:

    $ \ begin {align} х_1 + х_2 + х_4 = 0 \\ х_3-х_4 = 0 \\ 0x_2 = 0 \\ 0x_4 = 0 \\ \ end {align}

    $

    Обратите внимание, что последние два уравнения показывают, что все, что вы выберете для $ x_2 $ или $ x_4 $, даст вам однородное решение системы.

    Затем вы хотите найти зависимые переменные и задать параметры независимым переменным:

    $ \ begin {align} x_1 & = — x_2-x_4 \\ х_3 & = х_4 \\ x_2 & = s \\ х_4 & = т \\ \ end {align}

    $

    Наконец, подстановка независимых переменных $ x_2 $ и $ x_4 $ в уравнения зависимых переменных даст вам все решения однородной системы уравнений:

    $ \ begin {align} x_1 & = — с-т \\ x_3 & = t \\ x_2 & = s \\ х_4 & = т \\ \ end {align}

    $

    С помощью этих уравнений, учитывая любые два произвольных значения для $ x_2 $ и $ x_4 $, вы можете определить значения для $ x_1 $ и $ x_3 $, так что однородная система уравнений решается.

    Кроме того, обратите внимание, что вы могли бы решить уравнения для $ x_2 $ и $ x_4 $, сделав их зависимыми переменными и указав параметры $ x_1 $ и $ x_3 $:

    $ \ begin {align} x_2 & = — x_4-x_1 \\ х_4 & = х_3 \\ x_1 & = s \\ x_3 & = t \\ \ end {align}

    $

    Эти уравнения делают $ x_1 $ и $ x_3 $ независимыми переменными; и, при любом их произвольном выборе, вы можете найти зависимые переменные $ x_2 $ и $ x_4 $, которые будут решать однородную систему уравнений.Другими словами, операции со строками ничего не делают с зависимыми / независимыми переменными, вы выбираете, какие из них будут какими.

    систем линейных уравнений: исключение Гаусса

    систем линейных уравнений: исключение Гаусса

    Решать нелинейные системы уравнений довольно сложно, а линейные системы довольно легко изучать. Существуют численные методы, которые помогают аппроксимировать нелинейные системы линейными в надежде, что решения линейных систем достаточно близки к решениям нелинейных систем.Мы не будем здесь обсуждать это. Вместо этого мы сосредоточим наше внимание на линейных системах.

    Для простоты мы ограничимся тремя, максимум четырьмя неизвестными. Читатель, интересующийся случаем большего количества неизвестных, может легко развить следующие идеи.

    Определение. Уравнение

    a x + b y + c z + d w = h


    где a , b , c , d и h — известные числа, а x , y , z и w — неизвестные числа, это называется линейным уравнением .Если h = 0, линейное уравнение называется однородным . Линейная система представляет собой набор линейных уравнений, а однородная линейная система представляет собой набор однородных линейных уравнений.

    Например,


    и

    линейные системы, а

    является нелинейной системой (так как y 2 ). Система

    является однородной линейной системой.

    Матричное представление линейной системы

    Матрицы помогают переписать линейную систему в очень простой форме.Затем для решения систем можно использовать алгебраические свойства матриц. Сначала рассмотрим линейную систему


    Установите матрицы

    Используя матричное умножение, мы можем переписать линейную систему выше как матричное уравнение

    Как видите, это намного лучше, чем уравнения. Но иногда стоит решить систему напрямую, минуя матричную форму. Матрица A называется матричным коэффициентом линейной системы.Матрица C называется неоднородным членом . Когда , линейная система однородна. Матрица X — это неизвестная матрица. Его записи являются неизвестными линейной системы. Расширенная матрица , связанная с системой, является матрицей [ A | C ], где

    В общем, если линейная система имеет n уравнений с m неизвестными, то матричный коэффициент будет матрицей nxm, а расширенная матрица — матрицей nx (m + 1).Теперь обратим внимание на решения системы.

    Определение. Две линейные системы с n неизвестными называются эквивалентными тогда и только тогда, когда они имеют одинаковый набор решений.

    Это определение важно, поскольку идея решения системы состоит в том, чтобы найти эквивалентную систему, которую легко решить. Вы можете спросить, как мы сможем создать такую ​​систему? Легко, мы делаем это с помощью элементарных операций . Действительно, ясно, что если мы поменяем местами два уравнения, новая система все равно будет эквивалентна старой.Если мы умножим уравнение на ненулевое число, мы получим новую систему, по-прежнему эквивалентную старой. И, наконец, заменив одно уравнение суммой двух уравнений, мы снова получим эквивалентную систему. Эти операции называются элементарными операциями в системах. Посмотрим, как это работает в конкретном случае.

    Пример. Рассмотрим линейную систему

    Идея состоит в том, чтобы сохранить первое уравнение и поработать над двумя последними. Поступая так, мы попытаемся убить одного из неизвестных и решить два других.Например, если мы сохраним первое и второе уравнение и вычтем первое из последнего, мы получим эквивалентную систему


    Затем мы сохраняем первое и последнее уравнение и вычитаем первое из второго. Получаем эквивалентную систему

    Теперь мы сосредоточимся на втором и третьем уравнениях. Повторяем ту же процедуру. Попробуйте убить одного из двух неизвестных ( y или z ). Действительно, мы сохраняем первое и второе уравнение и добавляем второе к третьему, умножив его на 3.Мы получили

    Это, очевидно, означает z = -2. Из второго уравнения мы получаем y = -2, и, наконец, из первого уравнения мы получаем x = 4. Следовательно, линейная система имеет одно решение.

    Переход от последнего уравнения к первому при решении для неизвестных называется обратным решением .

    Имейте в виду, что линейные системы, для которых матричный коэффициент является верхнетреугольным, легко решить. Это особенно верно, если матрица имеет эшелонированную форму.Таким образом, фокус состоит в том, чтобы выполнить элементарные операции по преобразованию исходной линейной системы в другую, для которой матрица коэффициентов имеет эшелонированную форму.
    Используя наши знания о матрицах, можем ли мы как-нибудь переписать то, что мы сделали выше, в матричной форме, которая упростит нашу нотацию (или представление)? Действительно, рассмотрим расширенную матрицу


    Выполним над этой матрицей несколько элементарных операций со строками. Действительно, если мы сохраним первую и вторую строки и вычтем первую из последней, мы получим

    Затем мы сохраняем первую и последнюю строки и вычитаем первую из второй.Мы получили

    Затем мы сохраняем первую и вторую строки и добавляем вторую к третьей, умножив ее на 3, чтобы получить

    Это треугольная матрица, не имеющая эшелонированной формы. Линейная система, для которой эта матрица является расширенной, есть

    Как видите, мы получили ту же систему, что и раньше. Фактически, мы следовали тем же элементарным операциям, что и выше. На каждом этапе новая матрица была в точности расширенной матрицей, связанной с новой системой.Это показывает, что вместо того, чтобы писать системы снова и снова, легко поиграться с элементарными операциями со строками, и как только мы получим треугольную матрицу, напишем связанную линейную систему, а затем решим ее. Это известно как гауссовское исключение . Подведем итоги процедуры:

    Исключение Гаусса. Рассмотрим линейную систему.

    1.
    Построить расширенную матрицу для системы;
    2.
    Используйте элементарные операции со строками, чтобы преобразовать расширенную матрицу в треугольную;
    3.
    Запишите новую линейную систему, для которой треугольная матрица является связанной с ней расширенной матрицей;
    4.
    Решите новую систему. Возможно, вам потребуется присвоить некоторые параметрические значения некоторым неизвестным, а затем применить метод обратной подстановки для решения новой системы.

    Пример. Решите следующую систему методом исключения Гаусса.


    Расширенная матрица

    Мы используем элементарные операции со строками, чтобы преобразовать эту матрицу в треугольную.Мы сохраняем первую строку и используем ее для получения всех нулей в любом месте первого столбца. У нас есть

    Далее мы сохраняем первую и вторую строки и стараемся, чтобы во втором столбце были нули. Мы получили

    Далее сохраняем первые три ряда. Добавляем последний к третьему, чтобы получить

    Это треугольная матрица. Связанная с ним система

    Очевидно, что v = 1. Установите z = s и w = t , тогда мы имеем

    Из первого уравнения следует Используя алгебраические манипуляции, получаем
    x = — — с т .
    Собрав все вместе, у нас есть

    Пример. Используйте метод исключения Гаусса для решения линейной системы


    Соответствующая расширенная матрица

    Сохраняем первую строку и вычитаем первую строку, умноженную на 2, из второй строки. Мы получили

    Это треугольная матрица. Связанная система

    Ясно, что второе уравнение означает, что эта система не имеет решения. Следовательно, эта линейная система не имеет решения.

    Определение. Линейная система называется непоследовательной или переопределенной , если она не имеет решения. Другими словами, набор решений пуст. В противном случае линейная система называется согласованной .

    Следуя приведенному выше примеру, мы видим, что если мы выполним элементарные операции со строками над расширенной матрицей системы и получим матрицу с одной из строк, равной , куда , тогда система несовместима.

    [Назад] [Следующий] [Геометрия] [Алгебра] [Тригонометрия] [Исчисление] [Дифференциальные уравнения] [Матричная алгебра]

    С.O.S MATH: Домашняя страница

    Вам нужна дополнительная помощь? Пожалуйста, разместите свой вопрос на нашем S.O.S. Математика CyberBoard.

    Автор : М.А.Хамси

    Авторские права 1999-2021 MathMedics, LLC. Все права защищены.
    Свяжитесь с нами
    Math Medics, LLC. — П.О. Box 12395 — El Paso TX 79913 — США
    пользователей онлайн за последний час

    Метод Гаусса-Джордана — обзор

    Пример 2.2.3

    Обращение матрицы Гаусса-Жордана

    Метод Гаусса-Жордана основан на том факте, что существуют матрицы M L , такие что произведение M L A оставит произвольную матрицу A без изменений, за исключением

    (a)

    , одна строка, умноженная на константу, или

    (b)

    , одна строка заменена исходной строкой за вычетом числа, кратного другой ряд, или

    (с)

    чередование двух рядов.

    Фактические матрицы M L , которые выполняют эти преобразования, являются предметом упражнения 2.2.21.

    Используя эти преобразования, строки матрицы могут быть изменены (путем умножения матриц) теми же способами, которые мы могли изменить элементы определителей, поэтому мы можем действовать аналогично тем, которые используются для сокращения определителей по Гауссу. устранение. Если A неособое число, применение последовательности M L , т.е.е., M = (… ML′′ML′ML), может приводить A к единичной матрице:

    MA = 1 или M = A − 1.

    Таким образом, что нам нужно сделать, так это применить последовательные преобразования к A до тех пор, пока эти преобразования не уменьшат A до 1 , отслеживая продукт этих преобразований. Мы отслеживаем результат, последовательно применяя преобразования к единичной матрице.

    Вот конкретный пример. Мы хотим инвертировать матрицу

    A = (321231114).

    Наша стратегия будет заключаться в том, чтобы записать рядом друг с другом матрицу A и единичную матрицу одинакового размера и выполнять одни и те же операции с каждой, пока A не будет преобразована в единичную матрицу, что означает, что единичная матрица будет иметь изменен на A −1 .

Добавить комментарий

Ваш адрес email не будет опубликован. Обязательные поля помечены *